This site is 100% ad supported. Please add an exception to adblock for this site.

pathologypt3

Terms

undefined, object
copy deck
How do we distinguish hypokalemia due to increased renal excretion from that due to increased GI losses?
1. hx 2. Urinary K+ -High: increased renal excretion -Low: increased GI losses
What type of renal tubular acidosis is common in diabetics?
1. type 4 (hypoaldo or aldo resistance)
Define CCK
=chocystokinin =GI peptide hormone -Made by: 1. I cells duodenum and jejunum 2. CNS Fxns 1. Stimulate gallbladder contraction 2. relax sphincter of Oddi (surrounds ampulla where bile duct enters duodenum) 3. stimulate pancreatic enzyme secretion -via cholinergic pathway 4. inhibit gastric emptying 5. tropic effect on exocrine pancreas and gallbladder mucosa
Where are maltase, lactase, sucrase, and isomaltase found?
=enzymes that participate in carbohydrate digestion =oligosaccharide hydrolases -found along brush border of SI -highest concentration in mid jejunum and prox ileum
Define VIP
=GI peptide hormone =vasoactive intestinal peptide Located in: 1. parasympathetic ganglia in sphincters, gallbladder, SI -acts as NT Fxns: 1. stimulate water and electrolyte secretion by jejunum, ileum, colon 2. relax intestinal SM, including sphincter 3. stimulate pancreatic HCO3- secretion 4. intestinal vasodilatation
Define gastrin
=GI peptide hormone -made by: 1. G cells of duodenum 2. pyloric antrum Fxns 1. stimulate parietal cells to secrete HCl 2. Stimulate histamine release 3. stimulate pepsinogen secretion 4. increase gastric blood flow 5. promote contraction of circular muscle of stomach 6. trophic effect on gastric, SI, and pancreas mucosa
Define Motilin
=GI peptide hormone -Made by: 1. M cells 2. Enterochromaffin cells in duodenum and jejunum -Fxns 1. regulate migrating myoelectric(motor) complex
What is the fxn of pepsin?
=initiate protein digestion in stomach -synthesized and released from chief cells as pepsinogen (activated by H+ or by itself) -secretion stimulated by: ACh, gastrin, HCl
Define intrinsic factor
=factor necessary for absorption of vit B12 by terminal ileum -binds to B12 -produced by parietal cells of stomach
What is the fxn of enterokinase?
-activates trypsinogen by cleavage to generate active trypsin -enterokinase produced in duodenum -active typsin can go on to activate other proteases (including more trypsin) and phospholipases
What is the fxn of bile salts?
=ionized form of bile acids (produced from XOL in liver and conjugated to taurine or glycine) -Fxn: 1. enhance absorption of fat and fat soluble vitamins -emulsify fat droplets into micelles
Define secretin
=GI peptide hormone -Made by: 1. S cells of duodenum -produced in response to increased acidity of duodenal mucosa, FA Fxns 1. stimulate secretion of HCO3- fluid from pancreas 2. increase bile production 3. inhibit H+ secretion by parietal cells
Define GIP
=GI peptide hormone =gastric inhibitory peptide -Made by: 1. K cells of duodenum and jejunum -release stimulated by glucose, AA, FA in lumen of intestine Fxns 1. Stimulates pancreas to release insulin 2. inhibits gastric acid secretion
What's in bile?
1. water 2. bile acids/salts 3. XOL 4. bilirubin -product of heme degredation 5. phospholipids
Define somatostatin
=GI peptide hormone -Made by: 1. D cells -pancreatic islets, gastric antrum, intestine -also found in hypothalamus Fxns: 1.inhibits release gastrin, CCK, most other GI hormones 2. inhibits gallbladder contraction 3. inhibits pancreatic secretion 4. inhibits gastric acid and pepsinogen secretion 5. inhibits SI fluid secretion 6. inhibits ACh release from myenteric plexus and decreases motility 7. inhibits release of glucagon, insulin from pancreatic islets
Contrast alkalemia and alkalosis
Alkalosis =process that tends to produce decrease in H+ [] that causes increase in pH Alkalemia =the final increase in pH
Define acute renal failure
=abrupt loss of renal fxn secondary to a decrease in GFR -often reversible -clinical findings related to loss of 4 major fxns of the kidney
Name the 4 major fxns of the kidney
1. water and electrolyte metabolism 2. regulation of acid base 3. excretion of nitrogenous wastes 4. production of hormones -1,25(OH)2vitD -epo
True or false: the majority of cases who survive an episode of acute renal failure are able to recover full renal fxn
-true
Name the 3 types of acute renal failure
-classified on basis of decreased renal fxn 1. Prerenal 2. Intrinsic renal 3. Postrenal
Define Azotemia
=condition characterized by abnormal levels of nitrogen-containing compounds (urea, creatinine) or other wastes in the blood -usu due to insufficiency of filtering of blood by kidneys -3 subtypes (prerenal, renal, postrenal) -All 3 subtypes have: 1. decreased GFR 2. Increased BUN and serum creatinine
Define Uremia
-literally=urine in the blood -clinical syndrome associated with fluid, electrolyte and hormone imbalances where metabolic abnormalities develop in parallel with deteriorating renal fxn -can be seen w/: 1. Chronic renal failure 2. Acute renal failure 3. Later stages chronic kidney disease -there is no single uremic toxin that accounts for all the manifestations of uremia
What is the mechanism of renal failure in prerenal azotemia?
-condition outside kidneys cause decreased renal blood flow -> decreased GFR -Causes decreased RBF: 1. true intravascular depletion 2. redistribution of intravascular volume/low effective arteriolar volume 3. mechanisms 1 or 2 associated w/agents capable of modifying renal vascular resistances (drugs like NSAIDS, ACEIs)
Name some causes of prerenal azotemia
-All cause decreased RBF -> decreased GFR -> loss kidney fxn 1. True intravascular depletion -diarrhea, hemorrhage, dehydration, overdiuresis 2. redistribution of effective/low arteriolar volume -pancreatitis, CHF, advanced liver disease, nephrotic syndrome, edema, sepsis 3. mechanism 1 or 2 associated w/agents able to modify renal vascular resistances -NSAIDs, ACE Is when used to tx CHF, cirrhosis, nephritic syndrome, edema
How do NSAIDs cause prerenal azotemia
-by modifying renal vascular resistances -normal physiologic response: low circulating volume -> activation RAAS -> increased ATII -> ATII stimulates both direct vasoconstriction of renal vessels AND synthesis of vasodilatory PGs (PGI, PGE) that mitigate by vasodilating to get balanced normal renal vascular resistance -NSAIDs block the synthesis of vasodilatory PGs -> unopposed vasoconstriction -> increased vascular resistance -> decreased RBF -> decreased GFR -> abrupt loss renal fxn -> azotemia
How do ACE Inhibitors cause prerenal azotemia?
-they interrupt the normal physiologic response to activate the RAAS system and its effects in response to low circulating blood volume -ACE Is: block production of ATII -ARBs: block effects of ATII to directly vasoconstrict renal vessels -result is decreased glomerular capillary pressure -> decreased GFR -ACE Is cause prerenal azotemia in the setting of already intravscaular volume depletion or redistribution of volume/low arteriolar volume -condition is reversible if ACE Is are stopped
AT II has a more potent vasoconstrictor effect on the afferent or efferent arteriolar?
-Efferent arteriole -also vasoconstricts afferent arteriole but less potent effect
What is the most frequent cause of acute renal failure?
=acute tubular necrosis
Name some sites of injury and causes of renal/intrinsic azotemia.
1. Vessels -inflammation (vasculitis) -ischemia (XOL emboli, Renal art thrombosis) 2. Glomeruli -acute inflammation (acute glomerulonephritis) 3. Tubules -ischemia (acute tubular necrosis due to ischemia) -toxins -obstruction 4. interstitum -acute inflammation (interstitial nephritis 2nd to infection, drugs)
Name some substances that are toxic to renal tubules
1. Drugs -aminoglycosides (antibiotics) -amphotericin (antifungal) 2. Radiocontrast (CT) 3. Endogenous substances -myoglobin -hemoglobin
Define Acute tubular necrosis
=kidney disorder resulting from damage to the tubular cells that can result in acute renal failure -Causes: 1. lack of oxygen 2. toxins -results in decreased GFR -> renal failure
Name 3 mechanisms for how Acute tubular necrosis decreases GFR
1. intratubular obstruction due to sloughed cell, cell debris or cell casts 2. backleak of tubular filtrate across epithelium 3. primary reductions in GFR mediated by activation of tubuloglomerular feedback -these 3 mechanisms prob work together to decrease GFR and cause renal failure
What is the mechanism of postrenal azotemia?
-obstruction along the urinary tract (renal pelvis -> urinary meatus) -causes of obstruction 1. stones 2. BPH 3. tumors 4. neurogenic bladder 5. pregancy 6. clots
Define oliguria
=decrased urine flow rate -anuria=no urine flow -sign of renal failure
True or false: the presentation of acute renal failure is variable
-true -may be asymptomatic w/increased BUN and creatinine -may have oliguria/anuria -may show signs of the precipitating cause -may show complications of uremia
Name the major clinical features of acute renal failure and how they relate to the major renal fxns
1. Disruption of fluid and electolyte balance -volume overload -hyperkalemia -hyperphosphatemia 2. Disruption acid/base -metabolic acidosis 3. Loss of excretion of nitrogenous wastes -uremia 4. decreased production of hormones -loss epo ->anemia -loss 1,25(OH)2vitD -> hypocalcemia, 2nd hyperPTH
Describe the effects of uremia on the: a.brain b. cardiovascular c. GI d. lungs e. immune system f. hematologic system
**no single uremic toxin accounts for all these effects a. Brain -asterixis, somnolence, seizure, coma 2. Cardiovascular -pericarditis, HTN 3. GI -nausea,vomiting, GI bleed 4. lung -pleuritis,pulmonitis 5. immune system -increased risk infections (UTI, pneumonia, sepsis) 6. Hematologic -bleeding diathesis, coagulopathies
How does ATN differ from prerenal and postrenal azotemia?
-prerenal and postrenal failure are rapidly reversible once condition causing renal failure is txed -ATN takes a couple of weeks for kidney to generate new tubule cells -1-2 period of oliguric for by period of increased urine output (poliuric) that decreases BUN and creatinine
True or false: prerenal azotemia can progress to acute tubular necrosis
-true -this is why early diagnosis is impt
A pt presents w/oliguria. Blood tests reveal elevated BUN and creatinine. How do you work him up?
1. complete hx and physical 2. bladder cath to rule out obstruction 3. labs (esp UA and urine indices) 4. Imaging test -u/s to rule out upper urinary tract obstruction
Name 5 factors in a pt's clinical hx that suggest intrinsic renal injury
1. Hx of prolonged prerenal azotemia 2. prolonged, severe HTN, perioperative periods, muscle crush injury 3. exposed to toxins, illicit drugs 4. exposure to nephrotoxic drugs -contrast 5. prolonged coma, seizures
Define FENa
=fractional excretion of sodium (Na) -determines percentage of filtered sodium that is excreted -use to help determine if acute renal failure is: 1. Prerenal (less than 1%) 2. Acute tubular necrosis (greater than 1%) -previous diuretic use will alter the FENa
WBC casts in the urine and a FENa greater than 1% suggest _________.
=acute tubular necrosis
RBC casts in the urine and proteinuria suggest _______.
=glomerulonephritis or vasculitis -need renal biopsy
Positive dipstick for presence of hemoglobin/pigment but RBCs absent in urine suggests ______.
=rhabdomyolysis or hemoglobinuria
How do we treat patients with acute tubular necrosis?
-there is no specific tx for patients with ATN -manage with dialysis until tubular tubular cells regenerate and kidney fxn is restored
Name the 4 indications for dialysis in patients w/acute renal failure
1. fluid overload (severe CHF or pulm edema) 2. severe metabolic acidosis 3. hyperkalemia 4.uremic syndrome
Name 5 adjuvant treatments for acute renal failure
-in addition to treating primary cause of failure (or dialysis for pts with ATN) 1. adequate nutrition w/protein, phosphate, potassium restriction 2. restore normal volume status 3. avoid administration nephrotoxic agents 4. avoid unnecessary catheters 5. administration of calcium and phosphate binders
What is the most common cause of death in patients of acute renal failure?
-infections -so you want to avoid unnecessary catheters -tend to die of infections during the recovery phase from renal failure
What is the leading cause of end stage renal disease?
=diabetes -Other caues 1. glomerulonephritis 2. HTN 3. cystic kidney
Define Chronic kidney disease
=slow, progressive, irreversible decrease in GFR -results from loss of functioning nephrons
How is chronic kidney disease classified?
-classified into five stages based on kidney damage and GFR -stages 1 (kidney damage but normal GFR < 90) to stage 5 (kidney failure with GFR <15 or dialysis) -as renal disease progresses to higher stage, increased risk of cardiovascular disease
How does the kidney compensate for decreased renal mass?
"hyperfxning nephron" =increased RBF and pressure to remaining nephrons -> increase GFR per nephron but maintains normal kidney fxn -the tradeoff is that these compensatory mechanisms can themselves produce injury (glomerulosclerosis) to the kidney that can cause disease progression even in absence of initiating insult -this mechanism acts along with other mechanisms to progress CKD
Describe the mechanisms that cause CKD to progress
-initial insult to kidney leads to: 1. compensatory increase in single nephron filtration and pressure -> glomerulosclerosis 2. Increased XOL, decreased TG -> mesangial prolif -> mesangial sclerosis 3. compensatory hypertrophy -> increased tubular oxgen -> increased generation of ROS -> interstitial inflammation and fibrosis CKD is progressed by multiple mechanisms that result in glomerular and interstitial damage
At what GFR do patients with CKD usu present with clinical signs of loss of renal fxn?
-GFR= 10-20ml/min
What are the changes to the following fluids, electolytes, substances as chronic kidney disease develops? 1. Na 2. fluids 3. potassium 4. phosphate 5. calcium 6. H+ 7. magnesium 8.urea
-CKD=progressive loss renal fxn 1. Increased FENa -w/decreased ability to adapat to changes in Na intake 2. fluids -loss of ability to concentrate or dilute the urine 3. Increased K+ secretion by remaining nephrons -also increased aldo promotes K+ excretion 4. decreased phosphate excretion b/c loss nephrons -> hyperphosphatemia 5. loss 1,25OHvitD produced by kidney, decreased Ca reabsorp -> hypocalcemia 6. decreased net excretion of H+ -> metabolic acidosis 7. decreased magnesium excretion 8. decreased urea excretion -> increased BUN
What molecule acts as a surrogate marker for uremic toxins in renal failure?
=urea -in general, the higher the BUN the more likely to present w/uremia sx -tolerance to uremic syndrome varies (acute renal failure uremia not well tolerated)
Name 3 factors leading to development of anemia in CKD.
1. Decreased RBC production -due to decreased epo production by kidney 2. shortened lifespan of RBCs during uremia 3. increased risk of bleeding during uremia
Name some major cardiovascular complications of CKD
1. HTN -almost 100% CKD pts develop 2. CHF 3. dylipidemia -> atherosclerosis/stroke 4.pericarditis
How does HTN develop in chronic kidney disease?
-loss kidney fxn -> can't excrete Na and water -> circulating volume expansion -almost 100% of CKD pts develop HTN -consequences 1. accelerated atherosclerosis 2. Left ventricle hypertrophy and CHF 3. worsening of renal failure
Name the 3 major contributing factors to CHF in chronic renal failure
1. volume overload 2. anemia 3. HTN
True or false: pregnancy is impossible in chronic kidney disease
-false -while pregnancy is rare some CKD pts get pregnant -few term pregnancies -pregnancy rare b/c uremia suppresses gonadal system -> 1. amenorrhea 2. inferility 3. decreased sperm count 4. decreased libido
Why do diabetics with end stage renal disease need less insulin?
-decreased insulin metabolism by proximal tubule -> increased insulin half life -> decreased need -uremia also associated w/insulin resistance
Name 2 common complaints in patients with CKD.
1. Halotosis (bad breath) 2. Pruiritis (itchy skin)
What is the major cause of death in patients with chronic kidney disease?
-cardiovascular complications -multiple mechanisms contribute 1. HTN 2. fluid overload 3. anemia 4. dyslipidemia (increased LDL and VLDL)
Name the 2 major strategies to tx chronic renal failure
1. Conserative measures -diet restriction -correct metabolic acidosis -BP control -tx of anemia -manage calcium phosphate 2. dialysis
True or false: diuretic therapy is ineffective unless coupled to sodium restriction
-true
True or false: chronic renal failure pts are at risk of developing hyperkalemia
-true -mechanism depends on cause renal failure -use of aldosterone antagonists also block K+ secretion in collecting duct -loop diuretics can help tx hyperkalemia -if refractory, may require: 1. dietary K+ restriction 2. kayexalate (ion exchange resin)
How are caclium and phosphate levels managed in chronic renal failure patients.
-tend to have hyperphosphatemia and hypocalcemia 1. Phosphate binders and calium salts (decrease GI abs phosphate and increase Ca absorp) 2. 1,25(OH)vit D (active form to replace since kidney can't make) 3. dietary protein restriction -limit phosphate intake
What is the single most important factor in chronic renal failure progression?
=BP! -lowering BP below 130/80 slows progression of renal disease -ARBs, ACE Is have additional effects to prevent progression of renal disease but BP must be controlled to be effective!
True or false: there are more type I diabetics with end stage renal disease than type II
-false -diabetic nephropathy can develop in type I or type II -risk factors 1. poor glucose control 2. poor BP control
What is an early sign of diabetic nephropathy?
=microalbuminuria -this can progress to proteinuria and development of chronic kidney disease/renal failure
Define cystic kidney disease
=major cause of endstage renal disease 1. Autosomal dominant mutation in ADPKD-1 gene on chromosome 16 2. onset of sx and develop renal cysts ages 20-25 3. Also develop HTN 4. Increased risk of UTIs, infections and episodic flank pain 5. Associated complications -cerebral aneurysm (10%) -mitral valve prolapse (25%) -liver cysts 6. tx is supportive (control BP, tx UTIs)
Describe hemodialysis
-tx for end stage kidney disease or acute renal failure if indicated -blood is removed from body -> chemical composition of blood modified by diffusion across semipermeable membrane -> modified blood returned to pt -usu 3x's/week -requires fistula/shunt for blood access -lots of complications (mainly cardiovascular)
Describe peritoneal dialysis
-place diasylate in peritoneum where it bathes peritoneal capillaries -give time for diffusion across capillaries -remove excess fluid (now full of toxins, wastes, etc) -can be done at home but need pts that are very careful b/c risk infections at catheter port
Name the major complications of kidney allograft
-due to effects of immunosuppressive drugs 1. infections 2. malignancies 3. cardiovascular disease 4. chronic rejection
What is the most common renal malignant tumor in adults? Kids?
Adults=renal cell carcinoma Kids=Wilm's tumor (nephroblastoma)
Who gets Wilm's tumor?
=most common renal tumor in kids (usu ages 2-5) -very common tumor of childhood -neoplasm of renal parenchyma -sporadic or familial -WAGR syndrome associated -deletion of short arm of chromosome 11 or loss of WT1 suppressor gene -usu present with palpable mass -prognosis is good -
Name 3 genetic syndromes associated w/Wilm's tumor
1. Denys-Drash syndrome -90% chance Wilm's tumor -WT1 gene deletion 2. WAGR -Wilm's tumor, aniridia, gonadal dysgenesis, retardation -WT1 deletion 3. BWS -loss of imprinting at WT2 locus ->gene overexpression
Describe the gross and micropathology of Wilm's tumor
Gross 1. large, tan solid mass Micro -classic trio of cells 1. metanephric blastoma 2. epithelial derivatives 3. stromal derivatives
Name 2 factors associated with poor prognosis for Wilm's tumor
1. presence of anaplasia =large hyperchromatic cells w/abnormal mitosis -can be focal or diffuse 2. involvement of extrarenal organs
Describe the clinical features of renal cell carcinoma
=major type of renal cancer in adults -onset 60s-70s -males affected more -can present as metastatic carcinoma at 1st present (freq mets: lung, liver, bone) -can have paraneoplastic syndromes -strong associations w/ 1. Von Hippel Lindau syndrome 2. smoking
A 67 year old man presents with hematuria, flank pain, and a palpable abdominal mass. He has a 40 year hx of smoking. What type of malignancy would you suspect?
=renal cell carcinoma
Name the 3 subtypes of renal cell carcinoma
1. clear cell -most common 2. papillary 3. chromophobe
Describe clear cell renal cell carcinoma
-most common subtype of renal cell carcinoma -mostly sporadic but can be familial -strong association w/Von Hipple Lindau disease -gross: yellow mass in renal cortex -can have areas of necrosis/hemorrhage in mass -micro: nests of uniform clear cells -likes to spread into renal vein, perirenal fat
Define Von Hippel Lindau disease
-tumor suppressor gene on chromosome 3p25 -mutations result in: 1.retinal and cerebellar hemangioblastomas 2. angiomas and cysts of viscera 3. renal cell carcinoma -if young person with RCC, suspect VHL
Angiomyolipoma is associated with what disease?
-tuberous sclerosis -neoplasm of renal cortex -mixture of fat, SM cells, and blood vessel walls -cure w/surgical excision
Which subtype of renal cell carcinoma is associated with the best prognosis?
=chromophobe -arise from intercalated cells of collecting duct
Gross path specimen of encapsulated tan nodule removed from renal cortex. It has a central scar. Microscopic exam reveals benign behavior with nests of uniform cells with pink cytoplasm. You suspect __________.
=oncocytoma
Describe the clinical features of a juxtaglomerular tumor
-seen in young people with HTN and hyperaldosteronism -tumor secretes renin
Urothelial tumors are tumors of what type of epithelium?
=transitional epithelium that lines the urinary tract -often w/painless hematuria -men more affected -age onset: 50-70 -can be either papillary or nonpapillary subtype -clinical stage most impt prognostic factor -tx: transurethral resection, cystectomy
What are some risk factors for bladder cancer?
1. Exposure to industrial solvents -beta napthalene 2. smoking 3. chronic cystitis 4. shistomiasis (parasite) 5. drugs (cyclophosphamide)
What type of malignancy has been associated with phenacetin abuse?
=transitional cell carcinoma in renal pelvis
Name some substances produced in paraneoplastic syndromes in renal cell carcinoma
1.epo -> secondary polycythemia Ectopic production of hormones 1. ACHT 2. gonadotropins 3. renin 4. parathyroid-like hormone 5. prolactin
Describe the sx of adult polycystic kidney disease
-autosomal dominant -onset usu 30s-40s 1. fullness of flanks -b/c enlarged kidneys 2. HTN 3. ureteral colic (blood clots from ruptured cysts), polycythemia, uremia 4. associated abnormalities: polycystic liver, cerebral aneurysm, cystic pancreas 5. progress to renal failure
Autopsy revealed gross specimen with enlarged kidneys with multiple grape-like cysts distributed throughout cortex and medulla. You suspect _______.
=adult polycystic kidney disease -bilateral involvement
Name some complications of adult polycystic kidney disease
1. hematuria and intrarenal hemorrhage 2. renal infection 3. nephrolithiasis 4. HTN 5. ruptured cerebral aneurysm tx: dialysis, kidney transplant
True or false: infantile polycystic kidney disease also has autosomal dominant inheritance
-false -it's autosomal recessive -most kids die during perinatal period or early childhood -multiple cysts at birth
Who gets acquired polycystic kidney disease?
=pts on hemodialysis for end stage renal disease -kidneys may be normal sized -high % of cysts have tumors arising from them -can present as intrarenal hemorrhage
A rare disorder in teenagers affecting the medulla of the kidneys with polyuria and polydipsia, salt wasting, cysts, and progressive renal failure
=uremic medullary cystic disease/ juvenile nephronophthisis -can also have hypocalcemia -> bone changes (ostetitis fibrosis cystica and hyperplastic parathyroid gland) -severe refractory anemia -small kidneys w/multiple medullary cysts
Renal disorder characterized by multiple medullary cysts, renal stones. No renal failure.
=medullary sponge kidney -autosomal dominant -usu asymptomatic
True or false: simple solitary renal cysts is a common lesion in adults.
-true -can be found on routine radiologic exam
Define urolithiasis
=process of forming stones in the kidney, bladder or ureter -characterized by claculi (stones) in urinary tract -increased incidence in men
Name 4 types of urinary calculi
1. Calcium stones -majority 2. ammonium magnesium phosphate stones =staghorn/sturvite stones 3. uric acid stones -secondary to gout or increased cell turnover (leukemia) 4. cysteine stones -associated w/cystinuria
Name 3 causes of hypercalciuria. What is the major consequence?
hypercalciuria 1. increased intestinal absorption of calcium 2. increased primary renal excretion of calcium 3. hypercalcemia -hyperparathyroidism -malignancy -other causes Consequence: increased risk stone formation
________ stones are radioopaque while ______ stones are radiolucent.
Radioopaque=calcium stones Radiolucent=sturvite/staghorn
Name 2 baacteria that cause ammonium magnesium phosphate stones
=sturvite/staghorn stones -formed in alkaline urine (urease) 1. Proteus 2. Staphylococcus
What ion determines ECF volume?
=Na -ECF volume is appox TBNa (total body sodium) b/c the 2 are tightly coupled
TBNa excess means you will be: a. euvolemic b. hypervolemic c. hypovolemic
=hypervolemic -excess ECF fluid volume -findings: 1. interstitial fluid -> pitting edema 2. intrvascular fluid -> elevated JVP
TBNa deprivation means you will be: a.euvolemic b. hypervolemic c. hypovolemic
=hypovolemic (b/c TBNa tightly coupled to ECF volume) -findings: 1. orthostatic hypotension 2. urine: decrease Na, increase uOSM, 3. increased BUN, creatinine
True or false: serum Na is a good measure of total body sodium
-false -it only reflects relative water balance
Localized edema (as opposed to more generalized edema) is most likely due to _________.
=infection/inflammation -not associated with excess TBNa and hypervolemia
Name the 3 normal determinants of Na excretion
1. GFR -decreased GFR -> decreased Na filtration -> decreased Na excretion 2. Aldosterone -increased aldo -> increased distal Na reabsorp -> decreased Na excretion 3. Other -humoral: increase Na reabsorption (renin, NE) -ANP increases Na excretion -peritubular capillary oncotic pressure -increased reabsorption
Name some conditions associated with secondary aldosteronism
=increased aldo due to increased activation of RAAS 1. Renovascular HTN 2. Hypovolemia 3. Certain Na retentive states -CHF -liver cirrhosis -nephrotic syndrome -drugs (vasodilators) -pregnancy
Name some causes of hypovolemia
1. Nonrenal causes -GI hemorrhage -skin/environmental losses -other blood loss 2. Renal causes -renal failure in general -excess exposure to diuretics -"salt wasting" nephropathy 1. medullary cystic disease 2. may require dietary salt to maintain BP -glucocorticoid def (Addison's disease)
How do we treat hypervolemia?
hypervolemia=excess ECF= excess TBNa 1. Diuretics 2. phlebotomy (old days)
What is the target and site of action of loop diuretics?
block Na reabsorption =furosemide/lasix Target: Na/K/2Cl cotransporter (decreases cotransporter) -most potent diuretics Site: loop of henle (medularry thick ascending limb) -all loop diuretics have same "ceiling"/efficacy but vary in potency and half life
What is the target and site of action of thiazide diuretics?
block Na reabsorption -target: Na/Cl cotransporter (decreases activity of transporter) -site: distal connecting tubule
What is the target and site of action of carbonic anhydrase inhibitors?
block Na reabsorption -target: carbonic anhydrase -> CA usu provides H+ needed for NHE (Na/H+ antiporter responsible for 2/3 Na reabsorption) Site: proximal tubule -not very useful diuretics b/c tendency to produce metabolic acidosis (b/c can't reabsorb HCO3-)
What is the target and site of action of amiloride?
block Na reabsorption -target: apical Na channel (ENaC) -> Na channel blockade -site: collecting duct
What is the target and site of action of spironolactone
target: aldo receptors on collecting duct cells -> aldo antagonist -> blocks aldo effects to increase synthesis of ENaC to promote Na reabsorption -site: collecting duct -impt for tx edema and ascites in hepatic cirrhosis -side effects: gynecomastia
When should edema be treated?
-if pulm edema -> tx quickly -if peripheral edema: also tx but more slowly with diet and diuresis
Name 3 consequences of diuresis
1. decreased circulatory volume -> decreased CO -> azotemia, weakness, fatigue, orthostatic hypotension, confusion 2. increased hypovolemic hormones -renin, aldo, NE, ADH 3. Electrolyte disturbances -decreased serum K+, increased serum HCO3-, decreased serum Na
Describe refractory edema (diuretic resistance)
-some Na retentive states (ex CHF) -increased Na intake -decreased or delayed intestinal drug absorption -decreased entry drug into tubular lumen -increased distal Na reabsorption (lumen dose vs natriuresis response curve is right shifted) -decreased loop Na delivery
What strategy is used to tx refractory edema?
-determine single diuretic dose which is effective (increase dose until get a diuresis response) 2. then think about giving dose more than once daily
Why are diuretics sometimes used in combo?
-b/c of braking (refractory after initial response to diuresis) -mechanism is increased distal downstream Na reabsorption -combos work at multiple sites of action along nephron -but be careful not to diurese too quickly (metabolic side effects more prominent)
How do loop diuretics, thiazides cause hypokalemia
-normally, as Na is reabsorbed in principal cells of Collecting duct, it generates a voltage gradient -> K+ leaves principal cell down the voltage grad -> decreased K+ -with diuretics-> block Na reabsorption in more proximal regions of nephron -> decreased fluid reasborption -> increased flow rate to distal nephron -> dilutes K+ [] in lumen -> increased K+ excretion
Define blood pressure
=hydrostatic pressure under which blood is contained in the arterial tree -pulsatile and highly variable -SBP: left ventricle contracting and forcing blood into arterial tree -DBP: left ventricle relaxing; pressure exerted on/by arterial vessels themselves =driving force behind circulation
True or false: mean arterial pressure is higher than systolic BP.
-false -it's in between diastolic and systolic =DBP + 1/3PP -PP=pulse pressure=SBP-DBP -b/c MAP=CO x SVR, BP can be increased by increases in either CO or SVR
Name 4 factors influencing BP
1. Age -increased age, increased BP (esp SBP) 2. Ethnicity -African Americans higher freq HTN 3. Time of day -BP higher during day 4. Level of activity -active, dynamic aerobic exercise increases SBP -isometric exericse (ex wt lifting) increases SBP and DBP
What is the purpose of taking BP measurements in: a. both arms b. lying and standing c. arm and leg
a. both arms -exclude vessel stenosis, aortic dissection b. lying and standing -exclude postural hypotension c. arm and leg -exclude aortic coarctation
What is the BP range for borderline HTN? For HTN?
Borderline 140-160/90-94 HTN >160/>95 usu want 2-3 blood pressure readings taken and averaged before a diagnosis of HTN should be made
True or false: 95% of cases of HTN are due to known secondary causes
-false -only 5% are secondary -95% are idiopathic or "essential" but are often familial
Name 3 organs damaged by HTN
1. heart -HTN is an independent cause of CV risk factors 2. Brain -increased risk of stroke 3. Kidneys
Name some causes of secondary HTN
1. Chronic renal disease -glomerulonephritis -polycystic kidney disease -renal insufficiency -renal failure 2. Renovascular HTN -renal artery stenosis 3. Pheochromocytoma 4. Primary aldosteronism -Conn syndrome 5. Coarctation of aorta 6. Thyroid disease -both hyper and hypothyroidism have been associated w/ HTN 7. Oral contraceptives
How could OCPs cause HTN
-happens in about 5% of women using OCPs over 5 yr period -Estrogens increase hepatic synthesis of renin substrate -blood pressure normalizes after stop taking OCPs
Name 2 causes of renal artery stenosis and who gets which cause.
Renal artery stenosis=2ndary cause of HTN -Can be due to: 1. atherosclerosis -older men 2. fibromuscular disease/dysplasia -younger women -kidney is tricked into thinking there is systemic hypotensin -> activate RAAS
How does Pheochromocytoma cause HTN?
=2ndary cause -adrenal medullary tumor of chromaffin cells -secretes lots of catecholamines (NE) -NE increases both CO and SVR as well as stimulating RAAS-> increased BP
How does Conn's syndrome cause HTN
=2ndary cause of HTN -adrenal adenoma secreting aldosterone -> increased Na+ reabsorption -> increased circulating volume -> HTN
In essential HTN, young persons tend to have increased _______ while older persons tend to have increased ________. (hint: MAP=COx SVR)
Young people (20-50) -increased CO -reflects increased sympathethic output -have elevated plasma NE levels relative to age matched controls Older people (60 and up) -increased SVR
Describe some of the pathophysiological findings in essential HTN
1. Young essential HTN: elevated plasma NE levels -suggest increased sympathetic nervous activity -> increased BP 2. elevated intracellular Ca and Na -increased Ca can potentiate heart and SM contraction 3. decreased urinary kallikrein excretion -diminished vasodilation 4. RAAS: no signs of excess activation 5. insulin resistance -high levels of insulin can provoke sympathetic outflow -most pts w/HTN obese 6. heritability -family and twin studies
Describe the mechanism of cardiovascular consequences of HTN
1. Heart failure -increased BP -> increased afterload -> LV hypertrophy -> heart failure 2. Angina pectoris, MI -increased BP -> accelerated coronary artery atherosclerosis -HTN increases risk of CHD death -Both increased SBP and DBP increase CV risk
Describe the mechanism of renal complications w/HTN
1. Nephrosclerosis -increased BP -> increased arteriolar sclerosis (scarring) -histopath evidence of both: 1. fibrinoid necrosis (severe HTN) 2. Afferent arteriolar sclerosis -can lead to azotemia, proteinuria
Describe the mechanism of CNS complications w/HTN
1. Thrombotic stroke -increased BP -> accelerated cerebral atherosclerosis 2. embolic stroke -increased BP -> increased carotid atherosclerosis 3. Hemorrhagic stroke -increased BP -> increased arteriolar aneurysms
Name some major risk factors for target organ damage (heart, brain, kidneys) w/HTN.
1. Severity of HTN 2. Presence of both SBP and DBP increases 3. Associated risk factors -serum XOL -cigarette smoking -male gender -ethnicity (African Americans sustain more renal damage) -genetic factors (positive fam hx of CVD) -younger age
Give some examples of HTN emergencies
1. Accelerated/malignant HTN 2. HTNsive encephalopathy 3. Acute aortic dissection 4. intracranial hemorrhage 5. severe epistaxis (nosebleed) 6. postoperative bleeding 7. MAO inhibitors + tyramine
Describe the sx and signs of a HTNsive crisis.
=usu DBP > 140 1. Fundoscopic findings -papilledema, hemorrhages, exudates 2. Neurologic findings -headache -confusion -somnolence -visual loss -focal deficits -seizures -coma 3. Cardiac findings -hear enlargement -CHF 4. Renal findings -oliguria -azotemia 5. GI findings -nausea -vomiting
Define malignant HTN
=DBP > 140 in adult -Associated w/: 1. Papilledema 2. rapdily progressive end organ damage -heart, brain, kidneys -poor prognosis (usu 6 months) -pathological hallmark: fibrinoid necrosis
Define Hypertensive encephalopathy
=cerebral dysfxn secondary to severe HTN (DBP > 140) -usu acute increase in BP in malignant HTN -papilledema almost always present -can have focal neuro deficits -global findings: headache -> confusion -> coma
Define accelerated HTN
=severe HTN (DBP 120-140) associated with retinal hemorrhages/exudates -if untxed is precursor to malignant HTN
What is the histopathological hallmark of malignant HTN?
=fibrinoid necrosis in the kidneys -seen as part of nephrosclerosis (scarring of the kidneys) along w/afferent arteriolar sclerosis
What causes HTNsive emeregencies?
-it varies from severe idiopathic HTN to secondary causes -if encephalopathy present, consider secondary HTN (esp renovascular HTN)
What are your goals when you evaluate a newly diagnosed pt w/HTN
1. Evaluate for target organ damage -heart, brain, kidneys 2. Check for causes of secondary HTN
Which HTN pts may need additional lab tests with a new diagnosis of HTN?
1. Younger pts -esp under 30 yrs 2. Pts w/"tipoff" clue suggesting secondary HTN 3. Severe or difficult to control HTN
While evaluating a newly diagnosed pt with acute onset severe HTN, you notice an abdominal bruint and evidence of azotemia. You suspect _________.
=renal artery stenosis -cause of secondary HTN -use MR angiogram to visualize the stenosis confirm with renal arteriogram (invasive) -can be treated surgically w/a stent
A pt comes in with episodes of headache, diaphoresis, and palpitations. He has noted feeling lightheaded when he stands up sometimes. He BP was not elevated when he was seen 2 weeks ago by NP but is today. You suspect ____________.
=pheochromocytoma -confirm diagnosis with UA to check for urine catecholamine metabolites -treatable cause of secondary HTN due to adrenal neoplasm secreting catecholamines -tx by surgical removal if unilateral -also get fam hx b/c pheochromocytoma can be associated with genetic syndromes (MEN II, VHL)
A newly diagnosed pt with HTN complains of muscle weakness and polyuria. His serum K+ is low. You suspect _________.
=Hyperaldosteronism (Conn's syndrome) -adenoma secreting aldosterone -confirm with tests to try to suspress aldo secretion 1. saline infusion -increased volume will suppress RAAS and aldo levels -if not supressed, suspect Conn's 2. give ACE I and aldo secretion not supressed, suspect Conn's -should also show decreased plasma renin activity -also do a CT to locate the tumor
What is the rule of 10's for pheochromocytoma?
-10% bilateral -10% extra-adrenal -10% malignant
True or false: Cushing's syndrome can be a cause of secondary HTN
-true =endogenous glucocorticoid excess
True or false: tx of HTN has been shown to be effective for severe HTN only.
-false -shown to be effective for mild and severe HTN -reduce risk of end organ complications (heart, brain, kidney)
What is the goal BP range with anti-HTN txs?
Systolic: 130-135 Diastolic: 80-85
Name 5 non-pharmacologic txs for HTN
1. Avoid NSAIDs or vasoconstrictors (decongestants) if possible 2. Avoid excessive EtOH consumption 3. Avoid excessive Na+ intake 4. Wt reduction if obese 5. exercise -sustained aerobic exercise 3 days/week -these may be enough to tx mild HTN -these are useful adjuvants in moderate-severe HTN
Name the 4 most important drugs used to tx HTN
1. Ca channel blockers 2. Diuretics -thiazide 3. ACE Is 4. Beta blockers Can also use alpha blockers
Name 2 important predictors of response to HTN monotherapy
1. Age -young people respond better to drugs that target the RAAS system b/c often sympathetic excess stimulation 1.Beta blockers 2. ACE Is -older people respond better to: 1. Ca channel blockers 2. diuretics 2. Ethnicity -African Americans tend to have more salt sensitive HTN that responds better than whites to diuretics -African Americans show poorer response to beta blockers
What proportion of people tend to respond to initial HTN monotherapy? What are some strategies to deal with this?
Only about 50% of people respond to initial monotherapy Strategies 1. Choose or add second drug with a DIFFERENT action 2. combo drugs to minimize side effects
Name some useful drug combos to tx HTN
1. Diuretic and beta blockers -beta blocker helps prevent hypokalemia caused by thiazide 2. ACE I and Ca channel blocker -greater decrease in BP when given in combo
How do thiazides work to lower BP?
Acute: inhibit Na reabsorption -> decreased plasma volume -> decreased CO -> decreased BP Chronic: decrease SVR -> decrease BP Dose-dependent side effects: -increased glucose -decreased sK+ -metabolic alkalosis -increased uric acid -increased LDL -thiazides can be 1st line or 2nd line drugs for HTN tx
How does clonidine work to lower BP?
=central alpha 2 adrenergic agonist -stimulates alpha 2 R in vasomotor center of medulla -> decreased sympathethic outflow -> decreased SVR -> decreased BP Side effects: dry mouth, sedation, rebound increase BP when drug is stoped
How does propranolol work to lower BP?
=beta blocker -multiple mechanisms of action 1. decrease Hr -> decrease CO -> decrease BP 2. decrease renin release (beta 1) -> decrease AT II -> decrease SVR -> decreased BP 3. central action decreased sympathethic outflow -> decreased SVR -> decreased BP 4. decreased NE release from sympathetic nerve endings (B2) -> decreased SVR -> decreased BP Watch out for contraindications
African Americans show a diminished response to monotherapy with all beta blockers except: a. metoprolol b. atenolon c. pindolol d. labetolol e. propranolol
=labetolol -possibly because blocks both alpha and beta adrenergic Rs
How does prazosin work to lower BP?
=alpha 1 adrenergic R blocker -postsynaptic alpha1 blockade -> decreased SVR -> decreased BP -dilates both arterial and venous blood vessels -side effects: first dose response (postural hypotension, syncope) -2nd line drug for HTN tx
How does Hydralazine work to lower BP?
=arteriolar vasodilator -open cell surface K channels -> membrane hyperpolarization -> dilates vascular SM of resistance vessels (arterioles) directly -> decreased SVR -> decrease BP -but very potent vasodilator so reflex increased sympathethic outflow -usu given in combo with other drugs (beta blocker, diuretic) to counter reflex sympathethic response or its effects -good for difficult to tx HTN
When is nitroprusside indicated?
=HTNsive crisis -exogenous source of NO -> vasodilates arterioles and venules -> decreased SVR -> decreased BP
How does captopril work to lower BP?
=ACE inhibitor -mechanisms 1. decreased ATII -> decreased SVR -> decreased BP 2. decreased aldo -> decreased Na reabsorption -well-tolerated first line drugs for HTN (even mild HTN)
How does losartan act to lower BP?
=ARB=Angiontensin II R blocker -competitive antagonist of ATII -> decreased SVR -> decreased BP -can be 1st or 2nd lines HTN tx, usu for pts that don't tolerate ACE Is
How does nifedipine, diltiazem, and verapamil work to lower BP?
=Ca channel blockers -decrease cytosolic Ca in heart and vascular SM -> decreased potentiation of muscle contraction -> decreased SVR and CO -> decreased BP -1st line HTN drugs -side effects: 1. nifedipine -flushing, headche 2. verapamil -conduction disturbances (AV block), heart failure
Name some contraindications to HTN tx with beta blockers
1. Obstructive airway disease -asthma, COPD 2. Heart failure 3. bradyarrythmias 4. Diabetics on insulin
True or false: HTN is more common in Caucasians than in African Americans
-false -more common in African Americans -also more likely to: 1. have severe HTN 2. have end organ damage 3. progress to end stage renal disease 4. have diminished responses to beta blockers EXCEPT labetalol 5. be more responsive to diuretics
Which of the following classes of drugs is most effective in slowing the rate of progression of renal failure toward end stage renal disease in pts w/ HTN: a. Ca channel blockers b. beta blockers c. ACE Is d. diuretics
=ACE Is
True or false: elderly pts with HTN often have isolated elevated SBP.
-true -due to decreased arterial compliance (b/c arteries stiffer) -> increased pulse pressure -> increased SBP
How long does it take the kidneys to respond to an increased H+ load?
=hours-days to increase H+ excretion by kidney -vs. 1. ECF HCO3-: immediate 2. Pulm increased ventilation (decrease pCO2): minutes-hours 3. ICF and bone buffering: 2-4 hrs
Define anion gap
=computational but not realgap -usu cations=anions in plasma in normal pt =[serum Cations] - [serum anions] =[serum Na] -([sCl-] + [sHCO3-]) -normal: 12 +/- 2 mEq/L -can use to evaluate pts with acidosis
Why is alkalosis and acidosis bad?
-b/c your enzymes work optimally at pH 7.4 -proteins may become denatured at pHs above or below -> disrupt biological fxns
Contrast acidosis and acidemia
Acidosis =process of generating increased H+ that causes decreased pH Acidemia =final decrease in pH
Describe the development of acidosis.
1. Primary/creation -gain H+ w/ associated decrease in HC)3- -results in decreased pH 2. Secondary/compensation -Pulm: increase ventilation -> decrease pCO2 -> increase pH -partially compensates -can calculate degree of expected compensation using nomogram w/ serumn HCO3- and pCO2 in pt
Where does the acid generated by our body come from?
-H+ is generated by the oxidation/hydrolysis of foodstuffs -about 1 mEq H+/kg/day
How does our body deal with acid generated daily to maintain physiological pH?
1. H+ buffering -Intracellular: proteins, phosphates, hemoglobin -Extracellular: HCO3- 2. Renal excretion
How does the kidney help maintain acid/base balance?
1. Reabsorb filtered HCO3- 2. Excrete additional acid -net acid excretion in urine= titratable acidity + NH4+ -urinary HCO3- -constituitive= titratable acid= HP04, creatinine, uric acid -inducible=ammonia -kidney can respond to systemic acidosis by increasing excretion of ammonia -ammonia grabs protons -> NH4+
What is the major extracellular buffer of H+?
=HCO3-
Name 2 sites on the nephron where acid is excreted.
1. Proximal tubule -Na/H antiporter -carbonic anhydrase generates the H+ for the transporter to reabsorb sodium -when increased excretion of ammonia, Gln in prox tubule is split into NH3+ and H+ 2. Cortical collecting duct -H+ ATPase
How can the anion gap be used to evaluate pts with metabolic acidosis?
-can narrow down the causes of acidosis -pts can be divided into 2 groups (each with different sets of causes of acidosis) 1. Pts with normal anion gap but increased Cl- 2. Pts with increased anion gap but normal Cl- -both groups of pts still show decreased HCO3-
Name some causes of acidosis with an elevated/wide anion gap
=anion gap > 15 mEq/L can be though of as: 1. Endogenous acids -renal failure -ketoacidosis -hypoxia -> lactic acidosis 2. Exogenous acids -ingested toxins forming acids -includes: methanol, ethylene glycol, salicylates, toluene -can detect by checking to see if measured osmolality does NOT equal calculated osmolality
What does MUDPALES mean?
=causes of acidosis w/elevated anion gap (> 15 mEq/L) 1. Methanol 2. Uremia 3. Diabetic ketoacidosis 4. Paraldehyde 5. Alcoholic ketoacidosis 6. Lactic acidosis 7. Ethylene glycol 8. Salicylates
Name 3 mechanisms to develop metabolic acidosis with normal anion gap by increased Cl- levels.
1. HCO3- loss/wastage -GI/diarrhea -urinary -renal (RTA, CAIs) 2. Administration of Cl- containing acid -ex cholestyramine 3. Impaired renal H+ secretion -distal RTA -hypoaldosteronism -early renal insufficiency
When is an urninary anion gap helpful?
=use to differential diagnose normal anion gap, hyperchloremic metabolic acidosis -Normal: between 0-50 (negative) 1.Metabolic acidosis w/ Abnormal UAG (positive) means impaired distal H+ and NH4+ excretion -distal RTA -aldosterone resistance 2. Metabolic acidosis w/normal (neg) UAG -suggestes normal distal H+ and NH4+ excretion -other clinical state (ex diarrhea)
How do you calculate a urine anion gap?
=u[Cations] -u[Anions] =u([Na] + [K]) - u[Cl] -normal: 0-50 (negative) -useful to differentiate hyperchloremic metabolic acidosis w/ normal serum anion gap
Define renal tubular acidosis (RTA)
=cause of hyperchloremic metabolic acidosis -normal serum anion gap -occurs when kidneys fail to excrete acids into the urine, which causes the blood to become too acidic -can be classified based on site of impaired H+ excretion 1. Proximal (type 2) 2. distal (type 1) 4. decreased aldosterone or resistance
Describe distal RTA
=type 1 renal tubular acidosis -hyperchloremic metabolic acidosis -impaired secretion of H+ into urine at distal nephron -not able to acidify urine, even in presence of acidosis -can be due to inherited defect or result of damage to distal nephron as part of systemic disease process (SLE, Sjorens, etc) -usu has low blood K+ -usu low fractional excretion of HCO3- (less than 3%) -can tx w/small amt of HCO3- to normalize serum HCO3- levels
Describe proximal RTA
=type 2 renal tubular acidosis -hyperchloremic metabolic acidosis -HCO3- reabsorption is impaired -> increased fractional excretion of HCO3- (> 15-20%) = HCO3- wasting -need to give a lot of HCO3- to normalize serum HCO3- levels -Can be inherited (ex Fanconi's syndrome) or acquired (some chemo drugs-ifosfamide, also acetazolamide (CAIs)) -may have normal or decreased plasma K+
Describe type 4 RTA
=renal tubular acidosis -hyperchloremic metabolic acidosis -due to decreased aldosterone or resistance -Dx based on aldo level -will have increased serum K+ -can be inherited or acquired -impt acquired causes 1. spironolactone 2. ACE Is/ARBs 3. certain antibiotics (ex trimethoprim) 4. heparin 5. NSAIDs 6. immunosuppressive drugs 7. disease processes that alter kidney structure and fxn (diabetic nephropathy, lupus, transplant rejection)
What is the urine osmolal gap (UOG)? When do you use it?
=(measured urine osmolality -calculated urine osmolality) -urine osmolality= 2(u[Na + K] + uGlucose/18 + UUN/2.8 -use when urine anion gap is abnormal but suspect increase in unmeasured anion excretion (DKA, toluene) -if UOG > 40: distal NH4+ production intact -UOG < 40: distal NH3/NH4+ production impaired
When is serum osmolality used?
-use when suspect exogenous acids as a cause of metabolic acidosis w/elevated serum anion gap -serum osmolal gap= (measured- calculated) -elevated serum osmolal (osmolal gap) > 10 mOsm/kg: increased low molecular weight solutes -ex mannitol, ethanol, methanol, ethylene glycol, etc
What dose change in anion gap/change HCO3- ratio mean for metabolic acidosis?
-with high anion gap acidosis, a rise in anion gap width parallels decrease in HCO3- -because > 50% of excess H+ is buffered by cells and not HCO3-, increased anion gap usu exceeds decrease in HCO3- -use it to differentiate whether you have a pure elevated anion gap metabolic acidosis (1:1 to 2:1) or a mixed acidosis -mixed acidosis can be: 1. Combined high and normal AG acidosis (ratio <1:1) 2. Concurrent met alkalosis (ratio > 2:1) 2.
How does acidosis cause hyperkalemia?
-normally H+ and K+ are exchanged across the cell membrane -with increased H+ []: more H+ goes ECF -> ICF -increased K+: more ICF -> ECF -increased serum K+ with non-organic acidosis (diarrhea, renal failure) less with organic acidosis (DKA, lactate, ingestions)
How do we treat acidosis?
Emergency= pH < 7.2 -need to tx quickly to prevent BP instability, arrhythmias 1. If can quickly treat underlying cause, do that 2. If tx underlying cause takes too long, just tx the acidosis (give NaHCO3 infusion over 4-8 hrs) -want to get bicarb levels between 10-12 mEq, pH to at least 7.2
Describe the development of alkalemia.
1. Primary/creation -lose H+ with associated increase in HCO3- -leads to increased pH 2. Secondary/compensation -pulm: decrease ventilation -> increase pCO2 -> decrease pH -hypoventilation provides partial compensation -expected degree of compensation is variable and can be calculated w/nomogram based on HCO3- and pC02 levels
True or false: the sx and signs of alkalemia are nonspecific
-true -includes sx secondary to: 1. volume depletion -> weakness, dizziness 2. hypokalemia -> muscle weakness, polyuria, polydipsia
How do we distinguish primary vs secondary causes of mineralocorticoid excess?
-mineralocorticoid excess can result in metabolic alkalosis via actions of aldosterone -distinguish primary (ex aldosterone secreting tumor) vs secondary (hypovolemia, CHF) by renin levels 1.Primary hyperaldo -renin levels are low -unstimulated increased aldo secretion 2. Secondary hyperaldo -renin levels are high -high aldo levels driven by increased activation of RAAS
Name 3 causes of secondary hyperaldosteronism
-can cause metabolic activation -due to increased activation of RAAS -> increased aldo 1. Renovascular hypertension 2. Hypovolemia 3. Certain Na+ retentive states -decreased "effective arterial volume" -CHF, hepatic cirrhosis, nephrotic syndrome -CHF, cirrhosis show edema, not signs of volume depletion as can be seen in metabolic alkalosis
How are the causes of metabolic alkalosis classified?
-based on whether they are: 1. Initiating factors -loss of acid (vomiting, NG suction, diruetics, aldo excess) -HCO3- excess (pica) 2. Sustaining factors -due to impaired renal HCO3- excretion -> HCO3- trapped -hypovolemia -Cl- depletion -hypokalemia
How do diuretics cause metabolic alkalosis?
-block Na reabsorption in LOH or distal tubule -> increased Na to collecting duct -> more Na+ reabsorbed in exchange for K and H -loss of more K+ and H+ (also reabsorb more HCO3-)
How does hypovolemia sustain metabolic alkalosis?
1. Hypovolemia causes increased renin -increased renin release -> increased ATII ->increased aldo -> increased renal HCO3- reabsorption (Tmax) 2. Hypovolemia causes decreased GFR -results in decreased filtered HCO3- -> more in serum
How does Cl- depletion sustain metabolic alkalosis?
-decreased Cl- sensed by macula densa 1. Increased RAAS -> increased aldo 2. Increased distal H+ secretion -b/c decreaseed gradient for Cl- secretion by H+ ATPase 3. increased distal HCO3- reabsorption
How does hypokalemia sustain metabolic alkalosis?
-decreased serum K+ results in less to be exchanged at cortical collecting duct for Na+ reabsorption -so more H+ is excreted distally in exchange for Na+ reabsorbed to maintain electrical neutrality
What is the easy way to differential diagnosis metabolic alkalosis?
=measure urine Cl- [] 1. Low urine Cl (less than 15) -chloride depleted -causes: vomiting, NG suction, diuretics -tx: give Cl (as NaCl) = saline responsive 2. Urine Cl high (over 20) -causes: mineralocorticoid excess, severe hypokalemia -tx: (saline resistant) if minerocorticoid excess, give spironolactone and tx cause of excess -if hypokalemia, give KCl
Name 4 nonspecific txs for metabolic alkalosis
1. Saline (NaCl) -give to chloride depleted pts (have low urine chloride) -monitor urinary and serum pH 2. KCl -give if saline resistant, hypokalemia, edematous 3. Acetazolamide =CAI -use in pt w/ edema, hyperkalemic 4. HCl -only give to pt if intolerant of NaCl or KCl (renal failure, heart failure)
Define Bartter's syndrome
=inherited defect of impaired NaCl reabsorption in loop of Henle or distal tubule -characterized by: 1. decreased serum K+ 2. decreased serum Cl- 3. increased serum pH (alkalosis) -but no hx of diuretic use (even though clinical picture looks just like they've been on diuretics their whole life) -normal BP -elevated renin and aldo -can present neonatally or in childhood -will see increased Cl- in urine
Name 2 ion transporters in the nephron that can be affected in Bartters syndrome
-can result from inherited inactivating mutations in either: 1. Na/K/2Cl in LOH 2. NaCl cotransporter in distal tubule -results in syndrome where they look like they've been on diuretics their whole life (hypokalemic, hypochloremic metabolic alkalosis)
Contrast hyponatriemia vs hypernatriemia
Hyponatriemia =serum Na [] less than 135-145 -concentration, does NOT tell you TB Na Hypernatriemia =serum Na [] greater than 135-145 these disorders are due to abnormalities in water metabolism
Define tonicity
=effective osmolality =particles which can lead to water movement across cell membranes -ex: glucose, Na (NOT ureaa) vs osmolality (includes all particles)
Which contains more water: muscle or fat? What's the implication?
-muscle -b/c fat contains less H20, body habitus affects TBH20 -usu TBH20 is 55-60% of total body wt
What does the body defend more: plasma volume or tonicity?
-plasma volume
How does the normal body respond to ingesting 2L of H20?
1. H20 absorbed from GI tract 2. H20 absorbed distributed throughout TB H20 3. Addition of H20 causes a decrease in plasma osmolality -> suppress ADH and thirst sensation 4. Kidney dilutes urine (excretes ingested load of H20)
What is the major site on the nephron where urine is diluted?
=thick ascending limb of loop of Henle -NaCL reabsorbed but NOT H20 -> generates free H20 -> fluid is more dilute Cortical collecting duct is minor site -ADH must be suppressed to ensure excretion of free H20
Name the 3 major requirements for urinary dilution
1. Delivery of filtrate to thick ascending loop of Henle 2. Salt extraction by Thick ascending limb of Loop of Henle 3. Water impermeability of collecting duct -no ADH -normal membrane permeability -pathologic states affect one of these 3 requirements
All of the following are sx of hyponatriemia EXCEPT: 1. headache 2. nausea/vomiting 3. confusion 4. muscle weakness 5. seizures/coma
=muscle weakness (sign of hypokalemia) -but can get neuromuscular irritability -sx correlate w: 1. severity of depletion 2. how rapidly depletion develops
How do you evaluate a pt with hyponatriemia
1. Eval serum osmolality -determine if hypertonic, isotonic, or hypotonic 2. Evaluate urine osmolality -usu will be elevated -> tells you kidneys NOT diluting normally 3. Evaluate fluid volume status -may have increased, decreased, or normal ECF volume 4. Follow with Urinary Na
How do we treat hyponatriemia?
-depends on the cause 1. Volume depleted -restoration of volume status with isotonic fluids 2. Volume overloaded -Water restriction, ACE Is, diuretics for Na overload (not for the hyponatriemia), ADH antagonists 3. Euvolemic pts -H20 restriction, hypertonic saline and loop diuretics (if severe); SIADH-drugs causing neprogenic DI (resistance to ADH)
Why are ACE Is helpful to tx hyponatremia in volume overloaded pts?
-ex. CHF 1. Less avid NaCl reasborption/H20 reabsorption 2. ACE I also blunts thirst response
Name 2 defenses of the body against hypernatremia. Which is most impt
1. Thirst -most impt 2. ADH
Name 4 stimuli for the thirst response
-defends against hypernatriemia 1. increased osmolality 2. decreased arterial pressure 3. decreased blood volume 4. Ang II
Name 4 mechanisms to develop hypernatriemia
1. Hypdipsia -not taking in enough H20 2. Solute gain -ingest of salt, infusion of hypertonic Na solutions 3. Pure H20 loss 4. Hypotonic fluid loss -most common -if associated with Na depletion as well-> can see volume depletion -cuases: diuretics, GI fluid loss, excessive sweating, burns
How do we evaluate a pt with hypernatriemia?
1. Assess volume status -elevated -> salt excess -not elevated -> insuff H20 intake and volume can be decreased or normal 2.Assess Uosm +/- UNa -use to distinguish in setting of fluid state whether renal or extrarenal cause
How do we treat hypernatriemia?
1. Tx volume status 2. Tx hypernatremia -H20 prescription (5DW) -amt and rate of H20 infusion depends on acuteness of hypernatriemia -in general no more than half of what is needed to be replaced should be given over 24 hrs
What is the normal range for serum K+?
=3.5-5.5 mEq/: -hyperkalemia= concentrations greater than this range -hypokalemia=concentrations less than this range
Contrast internal vs external potassium balance
Internal -reflects distribution between ICF and ECF/across cell membranes -helps determine serum K+ -affected by several chemical factors External balance =difference between K+ intake and K+ excretion -determines: 1. Total body K+ 2. partly serum K+
True or false: most K+ excretion is GI (ie lost in stool)
-false -most renally excreted/lost in urine
Name 5 factors affecting internal K+ balance
=affect distribution of K+ between ECF and ICF -K+ is primarily in ICF 1. Catecholamines -beta agonists -> ICF uptake 2. Insulin -promotes K+ ICF uptake 3. Body fluid tonicity -hypertonicity pulls K+ out of cells -ex: mannitol infusion, high blood glucose levels 4. acid/base status -buffering of excess H+ into cells causes K+ to come out of cells 5. Hormones -aldo has slight increase ICF uptake -major effect to increase K+ excretion (external balance)
How does overuse of beta adrenergic agonists to tx asthma cause hypokalemia?
-associated with overuse of inhalers -beta agonists promote ICF K+ uptake -> hypokalemia
How is serum K+ affected by acid/base status?
1. Acidosis -buffering of excess H+ into cells causes K+ to come out of cells -> increase serum K+ 2. Metabolic alkalosis -deficiency of H+ causes H+ to come out of cells and K+ to move into cells -> decrease serum K+
What is the major determinant of external potassium balance?
=renal excretion -in settings of increased serum K+ kidney is able to rapidly increase K+ excretion to lower serum K+ Other factors: 1. GI losses -hypokalemia: diarrhea, laxative abuse, vomiting, NG suction 2. K+ intake -normally increased K+ intake not a problem unless impaired renal fxn or other problem K+ homeostasis
What is the major control of K+ excretion in the kidneys?
=secretion by the late distal convoluted tubule and the cortical collecting tubule -this is the source of the K+ that gets into the urine -K+ secretion by distal nephron is highly regulated
Name some important factors regulating K+ secretion in distal nephron
-K+ renal excretion is highly variable and highly regulated -K+ renal secretion at distal nephron is impt to determine K+ renal excretion and K+ external balance 1. Serum K+ level 2. Aldosterone -presence of aldo -resistance of kidney to aldo 3. Distal tubular flow rate -increased flow rate -> increased K+ secretion 4. Acid/base balance Other factors 1. Drugs (K+ sparing diuretics) 2. Anion delivery 3. Na+ delivery
How do syndromes of hyperaldosteronism or hypoaldosteronism influence K+ balance
-increased levels of aldo increase distal nephron secretion of K+ -> impt to determine renal excretion and K+ balance 1. Hyperaldo -increased K+ secretion -> decreased serum K+ -both primary and secondary causes of hyperaldo 2. Hypoaldo -decreased aldo -> decreased secretion -> increased serum K+ -Diabetics often less responsive to aldo at distal kidneys (type 4 RTA) 3. Pseudohypo -due to tx with spironolactone (aldo antag) -also leads to increased serum K+
Which of the following is NOT associated with a increased distal tubular flow rate? a. volume expansion b. loop, thiazide diuretics c. primary hyperaldo d. osmotic diuretics e. metabolic alkalosis f. all are associated
=all are associated -increased distal tubular flow causes increased K+ excretion -> decreased serum K+ levels -conversely, decreased flow rates can result in K+ retention
True or false: chronic metabolic alkaloses show kaliuresis, low serum K+, and low total body K+
-true -b/c alkalemia increases uptake of K+ into tubular epithelial cells from peritubular capillary (removing K+ from blood)
True or false: most pts develop hyperkalemia because of a single defect in K+ homeostasis.
-false -usu have MULTIPLE defets
What can cause a spurious finding of hyperkalemia?
1. Hemolysis during blood drawing -pt does not have hyperkalemia but lab results will say he does Others 1. local venous hyperkalemia during blood drawing 2. pseduohyperkalemia -in high WBC states where K+ leaks out of WBCs
Name some causes of hyperkalemia due to abnormal INTERNAL K+ balance
1. organic acidosis 2. hypertonicity -ex glucose overtransfusion for hypoglycemia 3. Insulin def 4. drugs -cardiac glycosides (digitalis); succinylcholine; beta blockers 5. massive cell breakdown -hemolysis -rhabdomyolysis -burns -chemo of leukemia
Name 5 causes of hyperkalemia due to abnormal EXTERNAL K+ balance
1. Increased intake w/poor renal fxn -salt substitutes, etc 2. Acute renal failure 3. Chronic renal failure -adaptation occurs w/increased GI loss of K+ -use when GFR below 10-20 hyperkalemia develops 4. selective hypoaldosteronism -pts with DM 5. tubular not responsive to aldo-K+ sparing diuretics 6. certain renal diseases
What is the major consequence of hyperkalemia?
=risk of lethal cardiac arrhythmias -K+ causes abnormal depolarization and repolarization of cardiac myocytes -can try to block w/ Ca+ gluconate
What EKG findings are associated with hyperkalemia?
-increased risk of fatal arrhythmias 1. peaked T waves 2. widening of QRS 3. disappearance of P wave 4. sine wave pattern 5. ventricular fibrillation
Name 3 strategies to tx hyperkalemia
1. Antagonize depolarizing effects of hyperkalemia -Ca+ infusion -effects only last 20 min so also need to take other measures to tx hyperkalemia 2. Redistribution of K+ into cells -insulin and glucose, beta agonists, HCO3- 3. Elimination of K+ from body -Kayexelate (K+ binding resin) -diuretics -dialysis
Name 3 substances can cause redistribution of K+ into cells
1. Insulin -give w/glucose so don't cause hypoglycemia 2. Beta agonists 3. HCO3-
What is Kayexelate used for?
=cation exchange resin -exchanges Na and K in gut -removed K+ in stool -causes a lot of cramping and diarrhea but very effective agent
What is the most common cause of hypokalemia?
=diuretics
Name some causes of hypokalemia due to abnormal INTERNAL balance
1. alkalemia 2. cellular incorporation -tx megaloblastic anemia w/ folate or B12: making RBCs and taking up K_ 3. beta agonists
Name some causes of hypokalemia due to abnormal EXTERNAL balance
1. decreased intake -alcoholics, tea and toast diet, dirt/clay eaters 2. Increased renal excretion -elevated urinary K+ 1. Normotensive 2. HTN -diff causes for each 3. Increased GI excretion -low urinary K+
How do we distinguish hypokalemia due to increased renal excretion vs due to GI loses?
-look at urinary K+ -increased renal excretion -> increased urinary K+ -increased GI loses (diarrhea, laxative abuse) -> low urinary K+
Name some sx of hypokalemia
1. Muscle weakness 2. Paralysis -rare but reversible once correct hypokalemia 3. Hyporeflexia 4. U waves on EKG 5. potentiation of digitalis toxicity
How do we tx hypokalemia?
-If asymptomatic -> give oral KCl -If severe sx -> IV KCl
What are some causes of increased renal excretion of K+ in a normotensive pt?
1. osmotic diuresis 2. diuretic drugs -all that act at proximal sites to K+ secretion in distal nephron 3. Loss of gastric fluid 4. Bartter's (genetic defect where they look like they've been on diuretics their whole life) 5. metabolic alkalosis
Name some causes of increased renal excretion of K+ in a hypertensive pt.
1. Primary hyperaldosteronism -adrenal adenoma or bilateral adrenal hyperplasia 2. Secondary hyperaldosteronism 3. Cushing's syndrome -glucocorticoid excess
Describe the development of glomerulonephritis
1. immunological derangement (unknown) 2.Immunoglobulins and complement become deposited in glomeruli 3. C5a attracts leukocytes 4. Cell activation, cytokines, growth factors 5. Ultrastructural alterations to the glomerulus (inflammation, cell prolif) 6. Pathophysiological changes -> clinical features -protein leakage -> proteinuria, low albumin, edema -red cell leakage -> hematuria, RBC casts -increased renin -> HTN -impaired filtration -> renal failure
Name the 4 pathophysiological consequences of damage to the glomerulus in glomerulonephritis
1. Protein leakage -> proteinuria, low albumin, edema 2. RBC leakage -due to rupture of glomerulus -> hematuria, RBC casts 3. Increased renin -> HTN 4. Impaired filtration -> renal failure
Which glomerular disease shows a linear immunoflorescence pattern?
=diseases due to Anti-glomerular basement membrane (GBM) antibodies -linear immunosflorescence pattern b/c continuous distribution of IgG Anti-GBM along BM
Which glomerular disease shows a granular "lumpy bumpy" pattern on immunoflorescence?
=glomerular diseases due to immune complex deposition -also other mechanisms -what gets deposited varies in this case with the disease (ex IgA in IgA nephropathy)
What's in the granular deposits on the GBM in membranous glomerulonephritis?
1. IgG 2. C3
What is the Anti-GBM antibodies reacting to?
=some that is present in continuous distribution along the GBM -once deposited, the Anti-GBM antibodies react w/specific domain on type IV collagen in GBM -this lead to linear deposition of Ig and complement -> glomerular inflammation -> reduced GFR, glomerular hemorrhage
In what percent of cases do anti-GBM antibodies cross-react with other structures besides the GBM?
-50% of cases -mostly cross-react with alveolar basement membranes in the lungs (Good Pasture's syndrome) -cause severe lung injury -> reduced gas exchage and hemorrhage
How do immune complexes cause glomerular disease?
1. Ag-Ab complexes are formed in the circulation in response to Ag 2. circulating immune complexes precepitate w/in glomerular filter system 3. glomerular Ig and complement deposit in granular pattern on GBM 4. Glomerular inflammation and structural changes 5. Reduced GFR, protein leak
Name 4 sites where electron dense deposits can be seen with EM of the glomerulus
-different diseases affecting the glomerulus result in deposits in different places 1. Subepithelial -membranous GN -acute post-infectious GN 2. Subendothelial -MPGN type 1, proliferative SLE GN 3. Mesangial 4. Intramembranous
What does "minimal change" to the glomerulus mean?
-describes fusion of foot processes of epithelial cells on outside surface of glomerular basement membrane -lesion visible only by electron microscopy -this is due to dysregulation of proteins w/in podocytes -> podocytes lose their architecture and collapse/flatten out
What does "membranous change" to the glomerulus mean?
=thickening of glomerulus basement -2 subtypes 1. membranous GN 2. membranoproliferative GN
What does "proliferative change" to the glomerulus mean.
=hypercellularity -several distinct patterns -ex: 1. mesangial 2. crescent formation
What does "sclerotic change" to the glomerulus mean?
=fibrous (scar) tissue replacing all or parts of the glomerulus -sclerotic change can be: 1. Focal 2. Global -ex end stage of inflammation
Name 4 types of pathological injury to the glomerulus?
1. Minimal change -fusion of podocyte foot processes 2. Membranous change -thickening of GBM 3. Proliferative change -increase in cell # 4. Sclerotic change -scar tissue replacing parts of glomerulus
Contrast the following terms for glomeruli injury: 1. Diffuse vs focal 2. Global vs segmental
1. Diffuse vs focal -diffuse= all glomeruli affected -focal = some glomeruli spared 2. Global vs segmental -Global= all of a given glomerulus affected -Segmental=only part of a glomerulus affected
What are the 3 criteria for nephrotic syndrome?
1. Proteinuria >3.5 g/24 hr 2. Hypoalbuminemia 3. Edema
Define acute nephritic syndrome
=abrupt onset of reduced urine volume and reduced GFR -usu w/: 1. hematuria 2. +/- red cell casts 3. often HTN
Define rapidly progressive glomerulonephritis
=severe, often irreversible nephritic illness, progressing from normal to end-stage within weeks
Define chronic glomerulonephritis
=end stage form of any type of glomerulonephritis or nephrotic syndrome
Name the 7 primary forms of glomerulonephritis
Non-proliferative forms 1. Minimal change nephropathy 2. Focal segmental glomerulosclerosis 3. Membranous GN Proliferative forms 4. Membranoproliferative GN -diffuse prolif 5. Mesangial IgA GN =IgA nephropathy -focal prolif 6. Acute proliferative GN -includes post-strep -diffuse prolif 7. Crescentic GN -rapidly progressive GN -diffuse prolif
What is the cause of minimal change nephropathy?
=primary glomerulonephritis -most common cause of nephrotic syndrome in kids -can also be seen in adults -most cases are idiopathic -rare association with NSAIDs, lymphoma
Define minimal change nephropathy
=Nil disease -nephrotic syndrome with characteristic pathological lesion as fusion of podocyte foot processes after injury by nephrotoxic cytokine from T cells -usu idiopathic -can be seen in kids but onset can be at any time -course of disease is usu episodic w/complete healing in between relapses -responsive to steriods
True or false: the injury to the glomerulus in minimal change nephropathy is thought to come from IgG and complement
-false -thought to be due to nephrotoxic cytokine from T cells -> lesion to internal proteins of podocytes -> podocytes lose their architecture and flatten out -> fusion of podocyte foot processes -fusion can be seen on electron microscopy only
Describe the clinical features of minimal change nephropathy
1. Features of nephrotic syndrome only -edema, proteinuria > 3.5g/24 hrs, hypoalbuminemia -No: HTN, hematuria, acute impairment of GFR 2. Seen in young kids but age of onset can be anytime 3. Episodic course 4. Usu responds to steroids and clears in 2-8 weeks
Define Focal sclerosing glomerulonephritis
=primary GN disease characterized by focal, segmental accumulation of hyaline material in mesangial regions -this means only some parts of a given glomerulus are affected and that there are glomeruli in kidney that are spared damage -this is actually a bunch of diseases that all cause a common pathology -clinical features: 1. mainly seen in young adults 2. persistent proteinuria 3. up to 50% have nephrotic syndrome 4. many can have HTN +/- hematuria 5. Uses progressive to renal failure
How does Focal sclerosing glomerulonephritis develop?
-damage to podocytes -> hyaline material deposits (focal, segmental) in mesangial regions of glomerulus -also see deposits of IgM, C3 -Lots of different mechanisms to cause injury to podocytes: 1. Classic immune type -nephrotoxic circulating factor 2. Cellular/ collapsing type -toxic (pamidronate, Parvo B19, HIV) 3. Genetic defect in podocyte protein 4. Secondary -reduced renal mass, obesity, hyperfiltration injury/stress of glomeruli
What are the clinical features of focal sclerosing glomerulonephritis
=primary GN characterized by hyaline deposits on some parts of some glomeruli -lots of different things can cause this common pathology Features: 1. usu in young adults 2. Most have persistent proteinuria 3. Up to 50% nephrotic syndrome 4. Other features: HTN, +/- hematuria 5. Usu progress to renal failure w/in 10 yrs 6. Up to 40% may be helped by steroids, can also try to give ACEIs, ARBs
True or false: focal sclerosing glomerulonephritis can recur in transplanted kidneys
-true
Define membranous glomerulonephritis
=primary glomerulonephritis characterized by diffuse thickening of glomerular basement membrane on light microscopy -Other pathology: 1. Immunofluorescence shows granular deposits of IgG and C3 along GBM 2. electron microscopy shows subepithelial deposits w/ intervening spikes of glomerular basement membrane between deposits -99% of cases are idiopathic but occasional secondary cases -autoantibody to glomerular epithelial cell antigen -most have nephrotic syndrome w/bland urinary segment -1/3 progress to renal failure, 1/3 stay the same, 1/3 get better
How does membranous glomerulonephritis develop?
1. Make autoantibody to glomerular epithelial cell Ag 2. granular subepithelial deposits of IgG and complement along glomerular basement membrane 3. activation of complement and insertion of the C5b-9 complex into podocyte membrane causes glomerular injury 5. end injury is diffuse thickening of glomerular basement membranes w/spikes of membrane coming up between granular deposits -clinically results in a nephrotic syndrome -99% of cases are idiopathic
All of the following are considered secondary causes of membranous glomerulonephritis EXCEPT a. NSAIDs b. tumor c. hep B d. chronic infections e. lupus
=NSAIDs -rarely connected to minimal change disease
Define membranoproliferative glomerulonephritis
=primary GN characterized by: 1. Granular C3 deposition along glomerular basement membrane 2. Thickening of GBM + mesangial proliferation + 2nd BM generated on inside of deposits 3. Low serum C3 levels 4. Mixed nephrotic-nephritic illness that progresses to renal failure in 6-7 yrs and is NOT responsive to steroids
Contrast type I vs type II membranoproliferative glomerulonephritis
Type 1 -usu idiopathic (or 2nd to hep c) -subepithelial immune complex deposits -> complement activation -> C3 dposition Type 2 -intramembranous deposits =accumulation of glycoprotein complex in GBM -> activation of complement -> C3 deposition
True or false: type II membranoproliferative glomerulonephritis recurs in 100% of renal transplants
-true -suggests that there must by a systemic origin to the pathogenesis of glomerular injury -results in intramembranous deposits that are accumulations of glycoproteins from degraded proteins -> activate complement -> C3 deposition -> glomerular injury with mesangial prolif and glomerular basement membrane thickening
What is the characteristic finding on a blood test for membranoproliferative GN?
=low serum C3 -C3 deposition is so heavy along the glomerular basement membrane that it actually lowers serum levels of C3 -seen in about 80% of cases -seen in both type 1 and type 2
Describe the clinical features of membranoproliferative GN
=primary GN 1. Uncommon disease 2. Type I can be secondary to hep C 3. Age onset 5-30 in idiopathic cases 4. Mixed nephrotic-nephritic illness (nephrotic + hematuria) 5. Progression to renal failure in 6-7 yrs 6. Most have HTN 7. low serum C3 is characteristic 8. type II disease recurs in 100% of renal transplants 9. not responsive to steroid tx
Define IgA nephropathy
=Berger's disease -common primary GN -due to increased amounts of an abnormal IgA in circulation =benign recurrent hematuria syndrome -exacerbated by mucosal infections -progression to renal failure in 10%
Describe the histopathology of IgA nephropathy
-due to abnormal IgA precipitating/getting stuck in mesangium -> activates inflamm including complement -> C3 deposition 1.Granular deposits of IgA and C3 in mesangium 2. Nonuniform proliferation of mesangium 3. Normal blood levels of C3
Describe the clinical features of IgA nephropathy
=benign recurrent hematuria syndrome 1. Most common GN in world 2. Triggered by fever/mucosal infections 3. Usu presents w/ persistent hematuria and proteinuria 4. Progression to renal failure in 10% 5. Nephrotic cases have worse prognosis 6. no specific tx
Define post-infectious GN
=acute proliferative GN -acute, self-limited nephritic syndrome 2 weeks after an infection (usu strep pharyngitis or strep skin infection) -seen more often in children than adults -characterized by low serum C3 levels, large subepithelial deposis, neutrophil infiltrate -mediated by deposition of infectious Ag on glomerular BM
How does post-infectious GN develop?
1. trapping of infectious Ag of a nephritogenic strain in glomerular filtration slits 2. immune response -> granular subepithelial deposits of IgG and C3, proliferation of enodthelial and mesangial cells, neutrophil infiltrate -characterized by lower serum C3 levels -seen about 2 weeks after original infections (ex strep pharyngitis or strep skin infection) -seen more often in kids than adults
Describe the clinical features of post-infectious GN
Acute nephritic syndrome 1. hematuria 2. initially reduced urine volume 3. elevation of BUN and creatinine 4. low serum complement 5. usu self-limiting and recover in few weeks 6. onset approx 2 weeks post infections (ex strep pharyngitis, etc) 7. seen more often in kids than adults
Define crescentic glomerulonephritis syndrome
=rapidly progressive GN -severe injury of glomerular capillary tuft -> bleeding and fibrin deposition in Bownman's space -characterized by extracapillary proliferation forming crescent of cells in Bowman's space -prodromal fever-like sx followed by rapid loss of renal fxn (weeks) -3 immune subtypes that can mediate glomerular injury
Name the 3 immune subtypes of crescentic GN
=rapidly progressive GN 1. Anti-glomerular basement membrane antibodies 2. immune-complex 3. ANCA-associated -each has characteristic pattern on immunofluorescence -all result in severe glomerular injury -> hemorrhage and fibrin deposition -> inflammation and loss of renal fxn
Define Anti-GBM crescentic GN
=rapidly progressive GN due to Anti-GBM antibodies -These are autoantibodies against type IV collagen of glomerular basement membrane -result in linear IgG deposition on immunofluorescence -if Anti-GMB cross-react with alveolar basement membrane -> lung hemorrhage -> Goodpasture's syndrome -see rapid progressive loss of renal fxn due to glomerular injury -tx: 1. plasmapheresis 2. steroids -must tx before creatinine above 6 m/dl
Describe crescentic GN due to immune complexes
-cause of immune complexes can be idiopathic of due to secondary causes (SLE, RA, severe bacterial infection) -characterized by granular IgG deposition along glomerular basement membrane on immunofluorescence -tx includes steroids, cyclophosphamide, plasmapheresis
Define ANCA-associated crescentic GN
=ANCA=antineutrophil cytoplasmic antibodies -when ANCA found in serum they can be pathogenic and cause vasculitis of glomerular capillary tuft -characterized by "pauci immune" pattern on immunoflurescence (min amts granular IgG in mesangium) -ANCAs can cause crescentic GN w/out vasculitis -when vasculitis involved can be due to: 1. Wegener's granulomatosis 2. microscopic polyangitis -tx with cyclophosphamide, steroids, plasmapheresis
Describe the clinical features of crescentic glomerulonephritis
=rapidly progressive GN 1. Prodome of fever-like systemic sx 2. Progressive development of oliguria or anuria (rapid loss renal fxn) 3. Active urinary sediment (hematuria, red cell casts) 4. some proteinuria 5. usu HTN 6. recovery of renal fxn in rare
Name 3 things pts with nephrotic syndrome are at increased risk for.
1. Infections -b/c loss of antibodies in proteinuria 2. acute renal failure -esp if dry them out too much w/diuretics or give them NSAIDs 3. venous thrombosis
Define Henloch-Schonlein purpura
=systemic IgA nephropathy -Characterized by: 1. renal failure 2. abdominal pain 3. purpura 4. large joint arthtitis 5. low serum C3
How are primary and secondary glomerulonephritides alike? Different?
Alike -primary and secondary share immunological, histological, and clinical features of GN -renal disease is indistinguishable Different -cause: primary due to idiopathic causes; secondary associated with some other systemic disease, neoplasm, infection -sometimes renal disease can manifest before a systemic disease (ex cancer) -sometimes curing secondary cause will take care of the renal disease
C-ANCA is characteristic of:
=Wegener's granulomatosis -systemic vasculitis that can be a secondary cause of glomerulonephritis
P-ANCA is characteristic of:
-Microscopic polyangitis -systemic vasculitis that can be a secondary cause of glomerulonephritis
Describe lupus nephropathy
-GN secondary to DNA-antiDNA AB immune complexes from systemic lupus -5 subtypes of renal disease 1. Types 1,2,3: minimal, mesangial, or focal lupus nephritis -variable amt of granular deposits of immune complexes in mesangium 2. Type 4 -membranoproliferative changes -mesangial and subendothelial immune complexes -most common 3. Type 5 -membranous lupus nephritis -subepithelial granular immune deposits -responds poorly to steroids -can see coexist type 4 and type 5 in same kidney
True or false: kids usu recover from Henoch-Schonlein purpura but adults may get progressive renal failure
-true =systemic IgA nephropathy + skin purpura, large join arthritis, abd pain
A tumor can be a secondary cause of what type of glomerulonephritis?
=membranous glomerulonephritis -sometimes renal disease manifests first and find tumor later
AIDs nephropathy and heroin nephropathy can look like what type of glomerulonephritis?
=focal glomerulosclerosis -both types are secondary causes of progressive renal destruction AIDs -may get HIV replication in renal cells Heroin nephropathy -due to microparticulates in heroin depositing in mesangium
NSAIDs can be a secondary cause of what type of glomerulonephritis?
=minimal change GN -may take several months to resolve after stopping NSAIDs
Name some renal diseases caused by Hep B. Hep C.
Hep B -secondary cause of membranous GN -rare cause of polyarteritis nodosa (vasculitis) Hepatitis C -Secondary cause membranoproliferative type I -can also cause cryoglobulinemia (antibodies in blood that thicken/gel-like at cold temp) or cutaneous vasculitis
Define Alport's syndrome
=hematuria causing progressive renal failure and deafness -may also have ocular abnormalities -due to genetic defects in 2 or more of 6 genes for alpha chain of type 4 collagen
Describe congenitally thin glomerular basement membranes
=benign familial hematuria -seen in siblings -autosomal dominant -recurrent episodes of hematuria but rare to get renal disease -good prognosis
Is diabetic nephropathy a nephrotic syndrome or nephritic syndrome?
=nephrotic -can progress to renal failure -seen in 30-40% type I DM and 10-20% type II DM -usu 14-17 yrs between onset of DM and overt proteinemia -can monitor development with microalbuminuria -pathology= diabetic glomerulosclerosis
Describe pathology of diabetic nephropathy
=diabetic glomerulosclerosis -subtypes: 1. diffuse/global mesangial thickening with accumulation of hyaline -also GBM thickening 2. Nodular (K-W) -mesangial lesions expand -> encroach on glomerular capillaries -> worsening renal fxn -problem exacerbated by HTN
How does diabetic nephropathy develop?
1. Hyperglycemia causes direct tissue damage -> glomerular sclerosis -> reduced nephron mass 2. hyperglycemia ALSO vasodilates renal afferent arteriole -> combined with HTN causes increased glomerular pressure -> hyperfiltration -> stress on remaining glomeruli -> glomerular damage -> loss of more glomeruli/nephrons
How do ACEIs/ ARBs help slow development/progression of diabetic nephropathy
-ACEIs/ARBs decrease systemic vascular resistance of efferent arteriole -> vasodilation -> decreases glomerular pressure -> releaves some of stress and damage on remaining nephrons
Name 4 things to help slow the progression of diabetic nephopathy.
1. Control hyperglycemia 2. Control BP 3. reduce dietary protein -also causes renal afferent vasodilation 4. ACEIs/ARBs
Describe renal amyloidosis
=chronic nephrotic illness due to amyloid accumulation within mesangial regions of glomerulus -secondary GN -amyloid deposits= indigestible fibrillar protein material -multiple myeloma can be a cause of amyloidosis -can identify amyloid deposits w/: 1. Congo red stain 2. apple green birefrigence in polarized light
True or false: HTN can directly cause glomerulosclerosis
-true -slowly progressive damage to glomeruli w/renal failure -African Americans esp at risk to develop glomerulosclerosis w/HTN
Define scleroderma kidney
=glomerulonephritis due to systemic disease -involves thickening of skin and arterial thickening -> damage to internal organs -characteristic "onion-ring" layer in arteries -in kidney results in: 1. reduced glomerular perfusion 2. hyperreninism 3. HTN 4. renal failure -HTN can be txed w/ ACEIs
Describe asymptomatic urinary abnormalities syndrome
Characterized by: 1. proteinuria or 2. hematuria Causes: 1. membranous GN 2. mesangial IgA GN 3. mild or early cases of other disease
Name 2 diseases associated with benign recurrent hematuria
=visible hematuria, episodic, often w/ persistent proteinuria Causes: 1. Mesangial IgA GN 2. Familial thin GBM disease
Define nephrotic syndrome w/ "bland" sediment
1. Proteinuria >3.5 g/day, lipiduria, hypoalbuminemia 2. Hypercholesterolemia 3. edema 4. +/- anasarca (extreme edema) Causes: 1. Membranous GN 2. Minimal Change disease 3. Focal glomerulosclerosis 4. Diabetic nephropathy 5. Amyloidosis
Name 3 causes of a mixed nephrotic/nephritic syndrome
=nephrotic syndrome w/ "active" sediment (RBCs and casts in urine) Causes: 1. Membranoproliferative GN 2. SLE 3. Henoch-Sconlein (systemic IgA)
Name 2 common causes of acute nephritic syndrome
Characterized by abrupt onset of renal impairment w/RBCs and casts in urine Causes: 1. Acute post-strep GN 2. SLE
Name 4 common causes of rapidly progressive nephritis
=crescentic nephritis -characterized by progressive nephritic syndrome over weeks -usu oliguric -crescents on biopsy Causes: 1. Anti-GBM GN 2. ANCA-associated GN (renal limited disease) 3. Systemic vasculitis 4. Severe cases of immune complex diseases (ex SLE)
Name 3 ways ACEIs/ARBs help ameliorate progression of renal failure
1. Reduce intracapillary glomerular pressure -> reduces glomerulosclerosis 2. Improved size-selectivity of GBM -> reduces proteinuria/lipiduria directly 3. Reduced proteinuria/lipiduria ameliorates tubular damage
What normally restricts the passage of proteins from filtration by glomerulus
-proteins restricted by: 1. size 2. negative charge of glomerular basement membrane, slit processes in between podocytes -this is esp impt to prevent loss of albumin (neg charged plasma protein)
Name 2 ways the kidneys normally prevent large loses of proteins in urine
1. Restriction of filtration of protein at glomerulus -restricted by: size (filtration slit processes in between podocytes) and negative charge of glomerular basement membrane 2. reabsorption of most filtered proteins at proximal tubule -albumin, low MW proteins
Normal amount of protein in urine is usu less than ________.
=150 mg/day -Includes: 1. filtered plasma proteins 2. glycoproteins secreted by uroepithelial cells -Tamm-Horsfall protein, IgA
Name 4 ways you could get increased amts of protein in urine
1. Loss of filtration restriction at glomerulus due to loss charge/size filter 2. Tubular dysfxn causes reduced reabsorption of protein 3. Too much protein that overwhelms capacity of proximal tubule to reabsorb 4. increased uroepithelial secretion -ex inflammation/infection causing increased IgA secretion
How can you get a dipstick negative for proteinuria but a high protein level measured by other means
-b/c dipstick is relatively selective for albumin -can get a false negative dipstick w: 1. very dilute urine 2. high concentration of immunoglobulins 3. high concentrations of Bence Jones protein -ex Multiple myeloma 4. high concentrations of mucoproteins
What can you use to substitute a 24 hr urine collection to quantify proteinuria?
=protein/creatinine ratio -normal: less than 0.2 -nephrotic proteinuria: ratio greater than 3.5
How do we evaluate a patient with proteinuria from a dipstick test?
1. Quantify the proteinuria 2. Check for evidence of renal or systemic disease -yes -> work it up 3. If no renal or systemic disease found, repeat dipstick 2-3 more times 4. If dipstick continues to be positive, quantify levels 5. If low grade persistent proteinuria (1-2 g/day), check for presence of orthostatic component and keep an eye on it
Name 4 consequences of nephrotic syndrome
1. Increased risk of infection -due to loss Ab in urine -also edema -> skin breakdown -> loss of normal barrier against infections 2. Increased risk of venous clot -b/c loss coagulation factors in urine 3. Increased risk of malnutrition -b/c loss plasma proteins carrying vitamins, minerals, hormones 4. Increased risk of cardiovascular disease -loss of albumin stimulates liver to make more albumin and lipoproteins -> accelerate atherosclerosis
Describe the musculature of the esophagus
-2 layers 1. inner layer -circular muscle -contract: lumen narrows and obliterates 2. outer layer -longitudinal muscle -contraction: shortening of esophagus -motor efferents via vagus from 2 nuclei: 1. Nucleus ambiguus -proximal esophagus 2. Dorsal vagus nucleus -distal esophagus and LES
Where are Auerbach's and Meissner's plexi? What are their roles for the esophagus?
1. Meissner's plexus -submucosal -regulates secretion and peristalsis of muscularis mucosae 2. Auerbach's plexus -located between inner circular muscle layer and outer longitudinal muscle layer -regulates contraction of outer muscle layers
What makes up the lower esophageal sphincter? What is its fxn?
Made up of: 1. internal sphincter -muscles of the esophagus 2. external sphincter -muscles of the crural diaphragm -inner and outer sphincters anchored by phrenoesophageal ligament Fxn 1. prevent reflux of stomach contents back into esophagus -tonically contracted and transiently relaxes when food bolus approaches
What makes up the upper esophageal sphincter?
Made up of: 1. inferior pharyngeal constrictor 2. cricopharyngeus 3. cranial cervical esophagus -constantly contracted -separates esophagus from pharynx
Describe peristalsis
1. Muscle contraction behind the bolus to push it forward -mediated by Ach 2. Muscle relaxation in front of bolus -mediated by NO
Define Achalasia
=incomplete relaxation of LES and aperistalsis of esophageal body Features: 1. Sx: dysphagia 2. Cause: idiopathic 3. Epidemiology: rare, affects both sexes, bimodal age distribution 4. Resembles megaesophagus of Chagas disease (caused by T. cruzi)
Describe the clinical features of achalasia
-listed in order of most freq to least freq 1. Dysphagia -both solids and liquids 2. difficulty belching 3. chest pain 4. nocturnal regurgitation 5. aspiration 6. loss of heartburn
Transient LES relaxation is the most freq mechanism causing __________.
=GERD -this is a relaxation of the LES that occurs independently of swallowing -pts with GERD have increased freq of TLESrs, esp after meals
A hiatal hernia can result in what esophageal condition?
=GERD -hiatal hernia= upward displacement of the diaphragmatic hiatus into the thorax -due to thinning of paraesophageal membrane -top part of stomach can either slide up into esophagus (type 1) or up next to esophagus (type 2) -type 1 hernia is found in over 50% of pts with GERD
List 3 mechanisms for how a pt with a hiatal hernia could develop GERD.
1. Hernia acts as a resevoir for stomach acid 2. impairs emptying of esophagus leading to re-reflux of acid 3. loss of diaphragmatic inhibition of reflux of stomach contents
Define Barrett's esophagus
=metaplastic process in which squamous epithelium of esophagus acquires features of intestinal epithelium -characteristically in middle aged white males -increased risk of adenocarcinoma
True or false: the pylorus is chronically open and closes when a wave of muscle contraction approaches
-true
How do NSAIDs cause direct damage to stomach mucosa?
=common cause of acute gastritis -most NSAIDs are weak acids and at pH 2 can cross lipid membrane to get into epithelial cells -once inside the cells, NSAIDs become ionized and release H+ -they also decrease hydrophobicity of mucus layer allowing diffusion of acid from gastric lumen
True or false: the systemic side effects of NSAIDs are more impt than the GI side effects
-true -Inhibition of COX-1 -> 1. impaired synthesis of PGs -results: decreased plt aggregation, diminished GI mucosal integrity, affect renal fxn -PG also impt inhibitor of stomach acid secretion
Define H. pylori
=gram neg curved rod -motile -> can burrow through mucosal layer of stomach -Other factors: 1. adhesins 2. urease -hydrolyzes urea to ammonia and CO2 -raises pH of stomach -over 1/2 world infected -infection transmitted fecal-oral -major cause of peptic ulcer disease
How does H. pylori cause peptic ulcer disease?
-H. pylori does NOT invade gastric mucosal cells but can attach via adhesins -> recruits PMNs -> inflammation -gastric inflammation is generated in host response to H. pylori -acute infection -> chronic infection and inflammation -can result in asymptomatic infection or depending on location, gastric or duodenal ulcer -H. pylori infections found in 96% of duodenal ulcers and 75% of gastric ulcers
True or false: H. pylori infections can cluster in families
-true -transmission is fecal-oral -most common cause of peptic ulcer disease
How can H. pylori infection be diagnosed?
1. Stomach biopsy and look for the bacteria -sea-gull shaped -this is the gold standard -can also check to see if stomach pH is basic (due to presence of ammonia generated by H. pylori urease) 2. C02 breath test -CO2 is a byproduct of H. pylori urease
How can H. pylori cause duodenal ulcers?
-H. pylori causes impaired HCO3- secretion -> decreased duodenal acid neutralization -in this setting, inflammation can lead to ulceration
Predict the consequences based on site of H. pylori induced gastritis: a. antrum b. corpus atrophic c. nonatrophic pangastritis
1. Antrum -> duodenal ulcer -acid secretion NOT affected 2. Corpus atrophic gastritis -> intestinal metaplasia -> dysplasia -> cancer -can also get gastric ulcer 3. Nonatrophic pangastritis -> MALT lymphoma NOTE: you can have asympomatic H. pylori infections
Name the 2 most common gastric cancers
1. Adenocarcinoma -75% of all gastric cancers 2. lymphoma
Are the rates of stomach cancer worldwide increasing or decreasing?
-decreasing -due to tx of H. pylori infections -but stomach cancer still very common type of cancer worldwide -higher incidence rates seen with Asian- Americans, African-Americans, Latinos
Name the 2 types of gastric adenocarcinoma.
=most common type of stomach cancer Types: 1. Intestinal type 2. Diffuse type
Describe Intestinal type gastric adenocarcinoma
=subtype of gastric adenocarcinoma Features: 1. Tumor cells form glands 2. Focal lesion tha can be polypoid or ulcerated 3. preceded by metaplasia-dysplasia 4. Associated with hypochlorydia in 80-90% of cases 5. Linked to genetic and environmental factors (nitrates, etc) 6. Seen in pts over 50 7. Better prognosis
Describe diffuse type gastric adenocarcinoma
=subtype of gastric adenocarcinoma Features 1. Tumor infiltrates as single cells 2. Signet ring cells 3. Diffuse lesion with littinus plastica/ leather bottle appearance 4.Seen in younger pts with equal male: female 5. less clear genetic/environmental association 6. worse prognosis
Define hypochlorydia
=lack of acid -risk factor associated w/stomach cancer -associated with 85-90% of pts with intestinal type gastric adenocarcinoma
True or false: GI ulcers can become cancerous.
-false -but cancers can ulcerate and look like benign ulcers -duodenal peptic ulcers always benign but gastric ulcers can be benign OR malignant (ie an ulcerated cancer)
How are H. pylori infections treated?
=2 antibiotics + PPI -but pts often feel bad (diarrhea, nausea) so pt compliance is low -tx of H. pylori infection is curative for peptic ulcer disease -PPIs rapidly heal peptic ulcers by reducing acid secretion
A pt presents with a very tender abdomen (acute abdomen). Imaging reveals air under the diaphragm. Pt has a hx of peptic ulcer disease. You suspect _________.
=perforated ulcer (creates a hole in the stomach) -very tender stomach is from perotinitis that develops after
An elderly pt comes in complaining of hematesis and abdominal pain. She has a hx of peptic ulcer disease requiring tx in the last 6 months. You work her up for ___________.
=hemorrhage of peptic ulcer -complication -can present with either hematesis or melena (dark, stinky stools) -May need endoscopy or surgery to stop bleeding -elderly have higher risk of mortality if taken to surgery
A pt has a hx of peptic ulcer disease. He now complains of getting full easily and throwing up whatever he eats. You suspect _________.
=Gastric outlet obstruction -location: pyloric channel or duodenal bulb -can happen w/acute ulcer or chronic
What is the most common cause of acute diarrhea?
=infection -acute diarrhea= less than 3 weeks in length -2 types: 1. Food poisoning -rapid onset -due to preformed toxins 2. ingestion & replication of organism -requires incubation period
Name 3 bacteria that cause "food poisoning" diarrhea
1. Staph aureus 2. B. cereus 3. C. perfringens
Name some bacterial causes of acute diarrhea due to ingestion and replication of bacteria
acute diarrhea=less than 3 weeks 1. salmonella 2. shigella 3. campylobacter 4. E. coli 5. Vibrio 6. Yersinia 7. Clostridium difficile
Name some major causes of acute diarrhea
=less than 3 weeks of diarrhea -usu self-limiting and will go away on own 1. Bacterial infections -most common cause 2. Parasitic/protozoan infections 3. Viral infections 4. Drugs/medications -ask about changes to meds, OTCs/herbs, starting new meds -diarrhea is a common side effect -many antibiotics can cause mild diarrhea
True or false: antibiotics commonly cause mild diarrhea
-true -usu onset 2-3 days after start, lasts 5-10 days -prob caused by changes in colonic flora -if severe diarrhea -> colitis -think Clostridium difficile infection
How do we work up diarrhea in immunocompromised?
-immunocompromised susceptible to infections that immunocompetent people get AND that immunocompetent people do NOT get -need to consider atypical causes of diarrhea (CMV, cryptosporidia) -also want to tx acute diarrhea sooner (vs waiting 3 weeks)
Name 6 major causes of chronic diarrhea
-Chronic diarrhea: lasts longer than 3 weeks -need to consider other causes besides infections (major cause of acute diarrhea) 1. Infection -Giardia -E. histolytica 2. Inflammation -ulcerative colitis -Crohn's 3. Drugs -laxatives -antibiotics 4. Malabsorption 5. Endocrine disorders 6. Motility/post-surgical disorders
What is the major cause of antibiotic associated colitis?
=Clostridium difficile
Name some tx of diarrhea
1. Rehydration -salt, glucose, water 2. Nonspecific tx -kaopectate -bismuth salicylate 3. Antimotility drugs -opioid derivatives 4. Antisecretory drugs -octreotide 5. antibiotics
Why is gatorade an effective rehydrating agent
-because contains H20, electrolytes, glucose -Na+/glucose transporter on enterocytes is never blocked so giving glucose/Na with water increases H20 reabsorption b/c increased Na/glucose uptake
How does loperamide work?
=antimotility agent -opioid derivative -tx diarrhea by: 1. decrease peristalsis -main effect 2. increase absorption 3. inhibit secretion
How does octreotide work?
=somatostatin analogue -specific diarrhea tx -suppresses neuroendocrine hormone secretion -decreases motility -useful for: 1. carcinoid syndrome 2. VIPoma
Who may need antibiotics to tx diarrhea?
-most people won't need b/c diarrhea usu self-limiting -but antibiotics needed for at risk populations 1. pregnant women 2. immunocompromised 3. very young or very old 4. severe disease 5. certain infections not usu cleared by our immune system -C. difficile -Shigella -Giardia -Entamoeba histolytica
Define diarrhea
-greater than 200 g/day stool output -usu associated with increase in fluidity, freq, volume of daily stool output -represents increase in stool water content (normally 60-85% of stool content) -diarrhea occurs when overall reabsorption efficiency falls below 98% (only small changes needed for large loss of fluids in stool)
Define malabsorption
=pathological interruption of the normal physiological sequence of digestion, absorption, and transport of nutrients -may result from disease of many different organs
How is malabsorption classified?
-Best classification based on premucosal, mucosal, or postmucosal causes -premucosal=impaired digestion -mucosal=impaired digestion -postmucosal=impaired transport
Describe normal protein digestion
1. Stomach provides pepsin 2. Pancreas provides proteases that digest proteins in lumens 3. Small intestine (enterocyte microvilli) -absorb small peptides and AA via secondary active process -contain intracellular peptidases -protein digestion can be disrupted by pancreatic disease or enterocyte damage (def in pepsin not a problem)
Describe normal carbohydrate digestion
1. Salivary amylase initiates carb digestion 2. pancreatic amylase 3. SI enterocyte microvilli -brush border disaccharidases -b/c only absorbed as monosaccharides -absorbed by secondary active transport or facilitated diffusion -
What is a polyp?
=any kind of circumscribed bump arising from the mucosal surface -It can be: 1. cancerous 2. benign neoplasm -adenoma 3. localized inflammation 4. normal tissue 5. non-neoplastic -hyperplastic, hamartoma -polyp does NOT mean adenoma -but usu want to biopsy to check
Name some consequences of malabsorption: a. protein b. carbohydrate c. fat
1. Protein malabsorption -hyperproteinemia, edema -usu occurs in conjunction with malabsorption of other nutrients 2. Carb malabsorp. -osmotic diarrhea -colonic bacteria fermentation -wt loss 3. Fat -steatorrhea -vit def (A, D, E, K) -wt loss
True or false: only the absorption of fats requires intact villi
-false -intact villi required for the absorption of all nutrients -damage to enterocytes/villi -> panmalabsorption of macronutrients
Which is the only dietary macronutrient that can be measured directly? a. protein b. fat c. carbs
=fat -measured: 1. qualitative -sudan stain 2. quantitative -72 hr collection with 100g/day fat load -greater than 7 g of fat/day in stool is abnormal
Describe the H2 breath test
=indirect test of carbohydrate malabsorption -unabsorbed carbs in the gut are fermented by bacteria -> produce H2 -Test can assess for: 1. Lactose intolerance -increased late peak in H2 prod after carb load 2. SI bacterial overgrowth -early peak in H2 production b/c bacteria normally in colon now in SI
What is the purpose of the D-xylose test?
=indirect test of malabsorption -tests for injury to enterocytes -D-xylose is a sugar that is passively absorbed, gets into bloodstream, and excreted by kidneys -low levels D-xylose in either blood or urine suggest enterocyte damage
What does the bentiromide test do?
=it tests exocrine pancreas fxn -bentiromide is ingested an normally cleaved to metabolite PABA in lumen, which is then absorbed and excreted in urine -if pancreatic insufficiency, a reduced amt of PABA is excreted
Pancreatic exocrine fxn must be reduced by greater than _____% to cause malabsorption
=90%
Where do most non-neoplastic polyps occur in the GI tract?
=colon -usu occur sporadically -freq increases with age -Can be: 1. hyperplastic polyps -most
Define hyperplastic polyp
=benign (non-neoplastic) polyp -often found in: 1. Colon -don't show true hyperplasia 2. stomach -show true hyperplasia Features: 1. small 2. slow cell turnover w/aberrant differentiation (colon) 3. smooth little domes 4. can be single or multiple 5. can show microsatellite instability -> colon cancer
What's the significance of a hamartoma in the GI tract?
=another type of polyp -benign -focal overgrowth of tissue components native to the organ in which it occurs
Adenomas of the _______ are very rare while adenomas of the _________ are more common
1. Stomach -very serious b/c high malignant potential 2. Colon -adenomas very common in the colon
True or false: metastatic lesions are the most common type of liver cancer
true
Define ischemic bowel disease
=ischemic injury (infarction) to either large or small bowel -Lesions can be classified as: 1. occlusive 2. non-occlusive -ex cardiac failure -can be extensive (ex. lesion of celiac, SMA, or IMA) or focal (lesion of small end-artery) -anastomoses between 3 major supply trunks can help deal with insidious loss of one of the vessels -infarcts vary in degree of wall of intestine affected
Name 3 causes of occlusive ischemic bowel disease
1. arterial -atherosclerosis -thrombus/embolism 2. venous -hypercoaguable state -OCPs -polycythemia vera -volvulus 3. vascular pathology -vasculitis -compression -amyloidosis
Name some causes of non-occlusive ischemic bowel disease
1. Cardiac failure -hypovolemia (shock) -sepsis -drugs (vasopressors, digitalis)
Name 5 factors that protect the pancreas from autodigestion
1. Synthesis, storage, and secretion of proteases as inactive proenzymes 2. Sequestration of enzymes in cell inside zymogen granules 3. Activation of trypsin requires enterokinase, which is only found in duodenum 4. Protease inhibitors are found in pancreatic juice and prevent premature activation 5. high pancreatic pressure prevents reflux of activated enzymes back into pancreas
Name some causes of elevated amylase.
1. Pancreatic leakage -acute pancreatitis, chronic pancreatitis, pancreatic cancer, endoscopic retrograde cholangiopancreatography 2. Absorption of amylase from a peritoneal surface -ruptured stomach,intestine, intestinal infarction, ruptured ectopic pregnancy 3. Salivary gland lesion -mumps, sialadenitis, alcoholism, bulemia 4. Tumor production -ovarian, lung, esophagus 5. decreased renal clearance -renal insufficiency, macroamylasemia 6. Pregnancy
Define macroamylasemia
=amylase complexed to either an immunoglobulin or other large molcule -> too big to be filtered by renal glomeruli -cause of elevated serum amylase
Name the 2 major causes of acute pancreatitis
1. Alcohol 2. gallstones acount for 70% of acute pancreatitis
How does steatorrhea connect to the pancreas?
-steatorrhea=fat in the stool -may be due to pancreatic insufficiency -> lack of digestion of fat b/c insufficient levels of lipase -need more than 90% reduction in exocrine fxn before would start to show -can be sx of chronic pancreatitis
Define pseudocyst
=complication of acute pancreatitis -liquefied area of necrotic pancreatic tissue walled off by fibrous tissue -can enlarge later -it can: 1. spontaneously resolve 2. become secondarily infected 3. perforate (large cysts)
Name some causes of ischemic bowel disease
1. Oclusive causes -arterial throbmoembolism -atherosclerosis of major vessels (MOST COMMON cause) -venous thromboembolism -vascular pathology (vasculitis, etc) 2. Non-occlusive causes -these are more common -heart failure -shock -sepsis -drugs (vasopressors, digitalis)
What is the most common cause of ischemic bowel disease?
=atherosclerosis of at least 2 major mesenteric vessels -> arterial obstruction
Describe development of ischemic bowel disease lesion
Extent of injury depends on time course and severity of ischemia 1. Mucosa affected first -epithelial injury, capillary injury 2. Fluid loss, hemorrhage, ulceration 3. Involvement of submucosa 4. Involvement of muscularis mucosa -> loss of motility 5. Perforation -also risk of perotinitis
What's the difference between occlusive vs non-occlusive lesions in ischemic bowel disease?
Occlusive -segmental lesion distribution (ie in the area of distrubtion of that artery) -uniform lesion Non-occlusive -patchy lesions -extensive distribution -variable severity
How do we classify intestinal obstruction?
1. Mechanical obstruction -something blocking the lumen -intrinsic bowel lesions -extrinsic bowel (something squeezing the bowel) 2. Fxnal obstruction -paralytic ileus -chronic motility issues
Name some causes of intestinal obstruction.
1. Gallstones 2. Trichobezoar (hair ball) 3. intussusception 4. Stricture -result of fibrosis 5. cancer 6. volvulus
Define intussusception
=invagination of a proximal segment of bowel into a more distal segment -results in bowel obstruction -Occurs in: 1. pre-existing bowel pathology 2. most often seen in infants and kids
Define volvulus
=twisting of a portion of the GI tract around itself -often causes bowel obstruction -can also get squeezed off and cause ischemic injury -happens more often with large bowel than with small bowel
Where is mechanical bowel obstruction likely to occur? What are some major causes?
-mechanical obstruction likely to occur in SI (85%) -Causes: 1. adhesions (75%) 2. cancer 3. hernia 4. crohn's
What is the main cause of large bowel obstruction?
=cancer -vs small intestine (more likely to be obstructed mechanically vs the large intestine); major cause is adhesions
How do you diagnose a bowel obstruction?
1. Xray -fluid accumulation 2. PEX -hear slushing sounds
Name some causes of fxn bowel obstruction
-2 major categories 1. Paralytic ileus -post-surgery -inflamm (perotinitis) -ischemia -metabolic (hypokalemia) 2. Chronic motility -achalasia: damage to nervous system for musculature can relax -Hirschsprung disease: damage to gangion cells -> muscles can't loosen up
A pt present with RLQ abdominal pain, nausea/vomiting, and fever. She shows rebound abdominal tenderness and leukocytosis on CBC. You suspect:
-apppendicitis -pathogenesis: 1. obstruction of appendiceal lumen (fecalith) -> -increased luminal pressure -ischemia -bacterial infection -necrosis/inflammation
Define diverticulosis
=presence of multiple diverticula without inflammation =little pockets of mucosa/submucosa that herniated through muscularis mucosa -most common in older people -usu asymptomatic but can get inflamed or cause obstruction, bleeding -prevalent in industrial nations -mainly seen in left colon
Define hepatitis
=inflammation of liver -many causes: 1. Chemical 2. Infectious (viral, etc) 3. Metabolic 4. Autoimmune
Name some clinical presentations of viral hepatitis
=hepatitis can manifest in many different ways: 1. Asymptomatic infection -infection resolves, no sx 2. Carrier state -persistent, asymptomatic infection -can infect others 3. Acute viral hepatitis (clinical) 4. Chronic viral hepatitis -hepatitis for more than 6 months 5. Fulminant viral hepatitis -very severe form
What do you see on biopsy of acute viral hepatitis?
1. Diffuse liver cell injury 2. liver cell cords disarray 3. apoptosis 4. lymphocytic inflammation
How do we distinguish on biopsy acute hepatitis due to alcohol vs viral?
Alcohol hepatitis 1. Mallory bodies =aggregation of cytokeleton elements -fatty change -PMN infilitration Acute viral hepatitis -apoptotic bodies (acidophilic bodies) -no fatty change -lymphocytic infiltration
What's the difference between hep B and hep C on liver biopsy?
-not much clinical difference -hep C there is less extensive injury in liver
What are the findings on liver biopsy of chronic hepatitis
-purpose of biopsy is to assess progression of liver disease or if it is remaining static 1. Inflammation around portal tract -> fibrosis -lymphocytic infiltrate 2. Piecemal necrosis -necrosis of bits of liver parenchyma -apoptosis of cells 3. fibrosis -forming fibrous septa -> cirrhosis (macronodular) -once see fibrosis, need to do something for pt (instead of conservatively observing disease) These are common histologic changes seen with hepatitis due to various causes
Name 3 viruses that cause chronic hepatitis
1. Hepatitis B virus 2. Hepatitis C virus 3. Hepatitis D virus
Define Fulminant hepatitis
=acute severe hepatitis -not just limited to viral hepatitis -rapid progression to liver failure (2-3 weeks) -get violent destruction of hepatic tissue -not caused by EtOH alone -
Name some causes of fulminant hepatitis
=severe acute hepatitis 1. Viral -Hep A,B,C, D -herpes virus -adenovirus 2. Chemicals 3. Drugs -tylenol (overdose) -CCl4 4. Natural toxins -mushrooms
Describe the pathological findings of fulminant hepatitis
1. Liver wrinkles -less tissue underneath 2. Liver shrinks -b/c massive destruction of its contents These features are common despite cause of fulminant hepatitis Pathogenesis is unknown
Which test is most useful for assessing surviving fxnal tissue mass of the liver?
=prothrombin time -it's a protein w/short half life so it won't hang around long if there's liver problems -abnormal: prolonged PT
If you see jaundice and sky high elevations of AST/ALT, think ______.
=acute hepatitis -esp viral -lots of viral causes but esp those w/particular affinity for infecting the liver (ie Hep A virus, Hep B virus, etc)
Name the 2 major causes of fulminant hepatitis
=severe, acute liver disease 1. Viral (75%) 2. Drug toxicity (25%)
True or false: alcohol by itself in large enough doses is capable of causing fulminant hepatitis.
=severe, acute hepatitis -false -EtOH by itself unlikely to cause such injury
Name 2 ways that toxins can damage the liver.
1. Intrinsic hepatotoxins -if you take a particular dose, anyone will get liver damage (ex. acetaminophen, certain mushrooms) 2. Host-dependent hepatotoxin -only some people get sick (either due to hypersensitivity to the toxin or a metabolic idiosyncrasy) -unpredictable who will get sick -ex halothane
Define intrinsic hepatotoxin. Give some examples.
=chemical that if taken in a certain dose will predictably cause liver damage to any user -Examples: 1. Acetaminophen (tylenol) 2. certain mushrooms 3. carbon tetrachloride 4. selective intrinsic hepatotoxins -cytotoxic: tetracycline, methotrexate, anti-neoplastic agents, EtOH -others can cause bile stasis -you get a nonspecific histopathology with all these various intrinsic toxins
Is host dependent hepatotoxicity more common or intrinsic hepatotoxicity?
=host dependent -can be due to: 1. hypersensitivity -ex halothane, other medications 2. metabolic idiosyncrasy -these are unpredictable rxns (some people get them, some people don't) -vs instrinsic hepatotoxins, everyone will get liver injury if give the right dose (predictable)
Define NASH
=non-alcoholic steatosis =fatty liver disease in people who don't drink -Associated w/: 1. DM 2. obesity 3. insulin resistance 4. disorders of lipid metabolism -pathogenesis not understood well -similar pathology to fatty liver disease due to EtOH
Name the 4 types of diarrhea
1. osmotic 2. secretory 3. inflammatory 4. dysmotility -either increased or decreased
A pt presents with dysphagia and difficulty belching. He notes nocturnal regurgitation of food but loss of heartburn. You suspect __________.
=achalasia -due to incomplete relaxation of LES -characteristic dilated esophagus with bird beak on imaging
How does achalasia develop?
-inflammation -> degeneration of myenteric plexus -> loss of nitrous oxide synthase -> incomplete relaxation of LES -also get aperistalis of esophageal body -result is a dilated esophagus w/bird beak on imaging
Name some conditions associated with GERD.
1. pregnancy 2. hiatal hernia 3. alcoholism 4. Zollinger-Ellison syndrome 5. smoking 6. chronic use NSAIDs
What is the most effective inital tx for GERD?
=PPIs
Name some predisposing factors for gastritis
1. H. pylori 2. NSAIDs 3. Smoking 4. EtOH 5. Genetics 6. Stress (trauma, burns)
Name 4 complications of peptic ulcer disease
1. Hemorrhage 2. Perforation 3. Outlet obstruction
Name some factors associated with peptic ulcers
Factors causing gastritis 1. H. pylori 2. NSAIDs Others 1.smoking 2. Zollinger-Ellison -gastrin-secreting islet tumor of pancreas 3. Primary hyperparathyroidism 4. MEN I -b/c gastrin secreting pancreatic islet cell tumor
Describe some characteristics of duodenal ulcers
Features: 1. benign 2. Abdominal pain relieved by food or TUMS 3. High gastric output 4. More frequent than gastric ulcers 5. Strong association w/H. pylori infections
Name the 2 histologic subtypes of gastric adenocarcinoma. Which has a better prognosis?
1. Intestinal type -tumor forming glands -can ulcerate 2. Diffuse type -tumor infiltrates as single cells -signet cells -diffuse leather bottle appearance -worse prognosis
Name some risk factors for gastric cancer
-usu caused by chronic gastritis/inflammation 1. H. pylori 2. Fam Hx 3. AChlorydia =lack of gastric acid 4. Genetic syndromes Lynch, FAP, Peutz-Jehgers 5. Salt preserved meats, nitrates
True or false: gastric MALT lymphoma can regress after tx of H. pylori infection
=true
Diarrhea that is: a. watery b. increased stool osmolar gap c. improves with fasting This suggests ______.
=omsotic diarrhea
Diarrhea that is: a. watery b. normal stool osmolal gap c. does NOT improve with fasting
=secretory diarhea
Name some causes of osmotic diarrhea
1. Exogenous -major cause -milk of magnesia, diet foods, lactulose 2. Endogenous-congential -lactase def, pancreatic insuff 3. Endogenous-acquired -post-enteritis dissacharidase def, celiac disease, short gut syndrome
Name some causes of secretory diarrhea
1. Exogenous -laxatives, bacterial infections with preformed toxins 2.Endogenous -Bacterial endotoxins (V. Cholerae, E. coli, Campylobacter) -hormone secretagogues (ex VIPoma)
What is the major cause of osmotic diarrhea?
=osmotic agents 1. milk of magnesia 2. diet foods 3. lactulose
What is the major cause of secretory diarrhea?
=bacterial endotoxins -V. cholera, E. coli, Campylobacter jejuni
A pt comes in with a 2 day hx of fever and bloody stools. The stool sample shows fecal WBC. What type of diarrhea is this?
=inflammatory Major causes: 1. invasive bacterial infection 2. Inflammatory bowel disease (Crohn's)
What are the 2 major causes of inflammatory diarrhea?
1. Invasive bacterial infections -Salmonella, shigella, campylobacter, E. coli 2. Inflammatory bowel diseases (Crohn's, ulcerative colitis)
Name some causes of diarrhea due to dysmotility
Increased motility -hyperthyroidism -dumping syndrome -diabetes Decreased motility -hypothyroidism -diabetes -watery diarrhea, have to exclude secretory or osmotic diarrhea first
How do we classify malabsorption syndromes?
1. by location of the lesion -premucosal, mucosal, or postmucosal 2. by what is malabsorped -global vs specific (ex lactose)
Name 2 causes of premucosal lesions leading to malabsorption
1. Pancreatic insufficiency -loss of digestion enzymes 2. bacterial overgrowth -malabsorption through multiple mechanisms 1. bacteria deconjugate bile acids -> fat malabs 2. deconjugated bile acids toxic to enterocytes -> direct mucosal damage 3. bacteria convert nutrients into compounds our bodies can't use -characteristic early peak on H2 breath test for bacterial overgrowth
How do we diagnose Crohn's disease?
1. lab tests -Anti- TTG antibodies -HLA typing (HLA DQ2, DQ8) 2. Endoscopic biopsy -distortion of normal bowel architecture w/loss of crypts, inflammatory infiltrate -cobblestone appearance
Hyperaldosteronism can lead to what electrolyte abormality?
=hypokalemia =cause of HTNsive hypokalemia -hyperaldosteronism can be: 1. primary -adenoma, bilateral adrenal hyperplasia 2. secondary -renal vascular HTN
How does elevated plasma glucose cause hyponatriemia?
=increased plasma glucose pulls water out of cells -> dilute plasma Na -for every 100 increase in glucose, Na decreases by 1.6
Name 2 antihypertensive medications that should be avoided in diabetics. Why?
1. Thiazide diuretics -cause hyperglycemia 2. Non-selective B blockers -block B2 stim 1. decreased glycogenosis -> prolonged hypoglycermia 2. decreased K+ into cells

Deck Info

477

kwithers

permalink